Sei sulla pagina 1di 111

e to

PHAN I ToAN 10

,.

7

DE THI CHINH THlIC(*) MCN : loAN 10

Thai gian : 180 phut (khong k~ thai gian giao d€)

Cin 1: (2(])

Giai phuong trinh: (x + 1)~x2 - 2x + 3 = x2 + 1.

Cin 2: (2(])

Cho tam giac ABC co cac canh BC = a, CA = p, AB = c , B = 2A, C = 4A, ban kinh duong tron ngoai tiSp IA R. Tinh

T=R2(~+~+~).

abc

Ciu 3: (2(])

Cho tam giac ABC can tai A. Duong tron nQi tiSp tam giac ABC tiep xuc canh AB tai T, dirong thang CTc~t dirong tron tai K khac T. Gia su K

IA trung di~m CT va CT= 6.fi. Hay tinh dQ dai caccanh cua tamgiac ABC.

Cin 4: (2(])

Chirng minh voi rnoi gia tri nguyen ciia m, tan tai s5 nguyen n d~ n3 - 1 i n2 -:- 87n + m

ehia hSt eho 191. Cin 5: (2(])

Cho a, b, c > O. Chung minh:

a4 b4

---;======= + +

a4 + ~(a6 + b6)(a3 + c3)2 b4 +~(b6 + c6)(b3 -+ a3)~

4

-+= c <1

c4 + ~(c6 + a6)(c3 + b3)2 - .

(") Gidm thi kh6ng giai thich gi them

0,5d

HuaNG DAN GIAI

Ciu 1: (2d)

Giai phuong trinh (x+1)Jx2-2x+3 =x2 +1 (1). T~p xac dinh R.

D~t Jx2-2x+3 =t, voi t?12.

Khi do phirong trinh(l) tro thanh (x + l)t = x2 +1 (2). (2) ¢:>x2-2x+3·-(x+ l)t+2(x-l) =0

¢:>t2_(x+ l)t+2(x-l)=0

~[::~-(

V o~ t = 2, ta co J x 2 - 2x + 3 = 2 ¢:> x2_ 2x + 3 = 4

2 [x =1-fi

¢:> x - 2x - 1 = ° ¢:> .

x =1+12

ver t=x-l, taco Jx2-2x+3 =x-l

~ {:2--~X+3::X-l)2 ~ {;~: ~ x EP.

V~y ~p hop nghiem cua phuong trinh la { 1-12; 1 +12} .

Cau 2: (2d)

0,25d 0,25d

0,25d 0,25d

0,5d

_a_=_b_=_c_=2R sinA sinB sinC

R 1 R 1 R 1

~a 2sinA'1)= 2sinB'-Z= 2sinC

~ S = 1.( 1 + 1 + 1 ) 4 sin2 A sin2 B sin2 C

= !.(3+cotg2A+cotg2B+cotg2C).

Trong ~ ABC: cot gA.cot gB + cot gBC.tgC + cot gC.cot gA = 1 0,5d

Taco

0,5d

2

, , 2 cot g a-I d d '

va ta co cot g a = , 0 0

2cotga

cotg2a = 1 + 2cotga.cotg2a

~ S = {[3+ 3 + 2(cotgA.cotg2A +cotgB:cotg2B + cot gc.cotg2C) ]

0,5d

8

9

}.

1 '

="4[ 6 + 2( cot gAcot gB + cot gB.cotgC + cot gC.cot gA) ]

1

="4(6+2)=2. O,5d

Cau 3: (24),

K la trung di~m cua CT va L la ti~p di~m cua duong tron voi canh

BC. Th~ thi : CK = _!_ CT (*). Suy ra L trung di~m cua BC,

2

CL 2 = CK.CT = _!_ CT 2 , O,5d

2

hay a2/4 = 36, hay a = 12 (1).

Ap dung dinh licosin trong tam giac BCT , ta co:

CT2 = BT2 + BC 2_ 2BT.BC.cosB O,5d

~ 72 = a2/4 + a 2 - 144. cosB

3

~ ,cosB = "4 (2), do (1).

M~t khac, ap dung dinh Ii eosin trong tam giac ABC, ta co:

b 2 = C 2 + a 2 -2ca.cosB ~ cosB = al2b (3). O,5d'

TiI (1), (2) va (3), ta co:

(a, b, c) = (12, 8-, 8). O,5d

Cau 4: (24)

D~t P(x) = x3 - l1x2 - 87x + m.

Ta co: P(x) == (x + a)3 + b (mod 191)

<=> x3 + 3ax2 + 3a2 x + a3+ b == x3 - l1x2 - 87x + m (mod 191) O,5d

{3a == -11(mod 191) (1)

~ 3a2 == -S7(mod 191) (2) , O,5d

b == m -a\modI91) (3)

(1) ~ 3a == IS0(mod 191) ~ a == 60 (mod 191) => 3a2 == -S7(mod 191)

V~y vrn E Z, t6n tai s6 nguyen a, b d~ P(x) == (x + a)3 + b (mod 191). Nhan xet: 191 las6 nguyen t6 dang 191 = 3k + 2

P( i)==P(j)(mod 191)=> (i + a)3 ==(j + a)3 (mod 191) .

D~t u = i + a, v = j + a, thi

u3==y\mod 191)=> U3k ==y3k (mod 191)

'u3kv2 == y3k+2 == yI91(modI91) == vtrnod lvl) (dinh li Ferma) (4) O,5d

10

,.

~ y2 ==U3ky3 ==u3k+3 (mod 191)

~ U3ky2 ==u3k .u3k+3 == u3k+l.u3k+2 == U3k+l.uI91 == u3k+1.u(mod 191)

==u3k+2 == u191 == u(mod191). (5)

Til (4) va (5) suy ra u == vtrnod 191) ~ i == j (mod 191) .

V~y n~u vi.] E {1,2, ... , 191}: i:to j(mod 191) thi PO):to P(j) (mod 191)

Suy ra ten tai nE{I,2, ... ,191}saochoP(n)=191(modI91), hay P(n):191 O,5d V~y voi moi gia tri nguyencua m, luon tan tai s5 nguyen n d~ P(n): 191.

Cau5: (2d)

~(a6 + b6)(a3 + C3)2 =

=~(a6 + b6)(~6 + 2a3c3 + c6) = ~(a6 + b6)(a6 + 2a3c3 + c6) =~(aI2 + 2a3b6c3)+ (2a9c3 +b6c6)+a6b6 +a6c6

:?~a6b6 +3a6b4c2 +3a6b2c4 +a6c6 =~(a2b2 +a2c2)3 =a2b2 +a2c2 O,75d Suy ra:

O,25d

Tuong nr

O,5d

O,5d

Cong cac bdt dimg thirc tren, ta co:

a4 b4

----r============+----r===~========

a4 + ~(a6 + b6)(a3 + C3)2 b4 + ~(b6+ ~6)(b3 + a3)2

c4

+ ~1.

c4 + ~(C6 + a6)(c3 + b3)2

11

DE THI DE NGHI(*) MCN:TOAN10

Thai gian : 180 phut (khong k~ thai gian giao de)

TRUONG THPI CHUYEN LE out eON. QuANG TRJ

Giai phirong trinh :

~~ -x·,h-x2 =1-2x2.

(1)

Bili 2.

Xet cac s6 thuc khong am thay d6i : al' a2, ... , a, co t6ng bing 1.

D~t: SK = ak +ak+1 +ak+2 +ak+3 (k = 1;6) GQi M = Max{SI,S2,S3,S4,S5,S6}'

Hay xac dinh gia tri nho nhM cuaM

@

. Tim tAt ca cac ham s6 f: R ~ R thoa man dieu kien: .

f(X3 _ y)+ 2y( 3f2 (x)+ y2) = f [y +f( x)) \ix, Y E R. (1)

Bai 5.

A B 1 , . '

Cho tam giac ABC thoa man : tg·- tg - = -. Chung minh rang :

2 2 2 .

1:\'" k'" " .\ d' d" 'J. ABC " 1.\ . A . B . C 1

I./leU l~n can Va U e tam glaC vuong a: sm - sm - sm - = - .

2 2 2 10

HUONG DAN GUI

Bail.

1 c=: 1 (r.--2 )2

2 -xv1-x- =2' v1-x--x

(0) f)J thi ilJ nghi va huang dJn giai cua cae truang tham du ki thi

2 ~ 1

2x -v2.x--=O

2

V~y phuong trinh (1) co 2 nghiem : x] = ~.; x, = ~ ( J2 - J6).

4

Di~u kien d~ phuong trinh (1) xac dinh : 1- x2 ~ 0 ¢:> Ixl s I. Khi do (I)¢:> ~1"'I_x2 -xl=I-2x2.

Phuong trinh co nghiem ¢:> 1- 2X2 ~ 0 ¢:> Ixl ~ ~ . Ta co : 1- 2X2 ~ 0 =;. "'1- x2 ~ X do do :

(2) ¢:> "'1- x2 - X = J2( ,.It - x2 + X )(v'I- x2 -X)

["'1- X2 = X

¢:> J2 ( v'1- x2 + x) = 1

1

x=J2

J2

X=-

2

J2-J6

X=---

Bai2.

M = Max {Sp.S2,S3,S4,S5,S6}

=;.12M ~ 4S] +S2 +S3 +S4 +S5 +4S6 = 9

= 4 L:ai + a2 + 2a3 + 3a4 + 3a6 + 2a7 + as ~ 4 i=]

12

13 '"

Bai 3.

Thay y = x3 vao (1) ta co

f(O)+ 2x3 (3f2 (x )+X6) = f( x3 + f( x)) (2)

Thay y = -f(x) vao (1), suy ra

f ( x3 + f (x)) - 2f (x)( 3f2 (x) + f2 (x)) = f (0) . (3)

Tir (2) va (3) ~ 4f3 (x)-3f2 (x).x3 _x9 = 0 ;V'x .

. ~(f(X)_X3)(4f2(X)+X3f(x)+X6)=0 ;V'x (4)

Ro rang : 4f2(X)+X3.f(x)+X6 > O;V'x:;t:O. Tu(4)taduQ'c

f (x) = x3 ; V'x .

Thir lai, thoa man dieu kien bai toan. Dap s6 : f (x) = x3 •

Bai 4. <

Taco: S=p(p-a)tgA =p(p_b)tgB =~p(p-a)(p-b)(p-c)

2 2

. A B S2 P - C a + b - C

~tg-tg-=. =--=---

2 2 p2 (p - a )(p - b) p a + b + C

A B 1

tg-tg- = - <=> 2(a+ b)-2c = a + b+c <=> a+ b = 3c (1).

2 2 2

~ abc r 2

,Ml;lt khac : S = pr = 4R ~ p.abc 4R = S = p(p-a)(p- b)(p-c) (2).

(1)<=>.{P = 2c . Tir (2) suy ra ab~ = p2 -(a + bjp+ab

p-c=c 4R

r 2

~ (1- 4R)ab = 2c (3)

." . A . B . C 1 r 1 T' (3) S ab 20 2 '

Gla su: sm - sm - sm - = - ~ - = - . u ta co a = - c , cung

2 2 2 10· 4R 10 ... 9

voi (1) suy ra : a va b la cac aghiem cua phirong trinh :

e -3ct+ 20 c2 =0 .t =~c·t = 4 c.

9 '13'23

Coi r~ng a ~ b, ta co:

Cho bAt phirong trinh:

a+~I_x2006 + x2004 +16 ~2~x2004 +16. a+~I- x2006

Tim s6 thirc a Ian nhAt de bAt phirong trinh co dung hai nghiem.

l::!: ~ b' +c' =(~ c J +c' =Gc J =a' ~ MBCvuong tai A.

Nguoc lai, gia sir: LlABC vuong tai A. Ta co:

:2==2~+C2 (1){~3:~:)2=b2+C2 ~{::!f

s =..!. be = pr ~ b = 4r(do: p= 2c) c = 3r

2 Tit (1) ta co

{a = 2R r 2 ABC 2 ~ 5r= 2R ~ -=-~ 4sin-sin-sin- =-

a = 5r R 5 . 2 2 2 5

. A . B . C 1

~sm-sm-sm-=-.

2 2 2 10

(f)i~u khimg dinh cua bai toan duoc chirng minh).

TRUONG Quac HQC HUE. TP. HUE

Bail.

Bili 2.

Cho m,n lel hai s6 nguyen dirong. Giai phirong trinh:

sin2nx+ 1 =cos2nx+ 1 .

sin2m+l x cos2m+1 x

Bai3.

Cho I lel tam duong tron nQi tiep tam giac khong can ABC. M la trung diem rc, N lel trung diem cua All J lel trung diem M N. GQi x, y, z lel ba dirong thang l~n hrot di qua A,B,C vel m6i dirong thang chi a chu vi tam

14

15 ,.

giac ABC thanh hai phAn bing nhau. Chung minh rang 4 duong thang x, y, z va IJ cung di qua met diSm.

Bai 4.

Cho n la s6 nguyen dirong. Xet bang vuong 2n hang va 2n cQt. M6i 0 vi~t vao mot s6 trong ~p {1,2,3, ... ,4n2}, hai 0 khac nhau vi~t hai s6 khac nhau. Tim s6 N IOn nhAt co tinh chAt : voi moi each vi~t s6 nhir tren t6n tai mQt hang hoac mot cQt rna tren hanghoac CQt do co hai s6 p, q thoa man:

Ip-ql~N .

Hu'ONGDANGW

Bai 1.

, ( a + ~ 1 - X 2006 - ~ X 2004 + 16 r

Viet lai ~ ~ 0 voi x2 ~ 1 .

. a+ 1- x2006

N~u a <0 bAt phuong trinh a + ~l- x2006 < 0 vo s6 nghiem. N~u a ~ 0 ta co phuong trinh a = ~16 + x2OO4 - ~I- x2006 .

Ham s6 v~ phai la ham s6 chan. Ham s6 tang tren [0;1] do do phirong trinh co nghiern khi a thuoc [3, JU], suy ra gia tri 16n nhat cua a la JU , luc do hai nghiem la x = -I, x = 1.

Bai 2.

D~t u = sinx ,v =cosx : u.v if- 0 va u2 + v2 = 1. Ta co phirong trinh: u2n + +1' = v2n + -21 I'

. u,·m+ V rn+

N~u uv < 0 mQt v~ nho han 1 v~ kia 16n han 1: khong thoa man.

Ham s6 y = x2" + 2~1I+1 giam trenl -! ; 0). V~y n~u u ,v < 0 ; u -:;:. v x

thi khong thoa man.

Xet u,v >0. u:l= v. Ta co (u2n _v2n)(u.v)2m+1 = u2m+1 _v2m+1 hay:

(U2n-l+ V2n-l+ uV(U2n-3+ V2n-3)+ ... + (UV)"-I(U+v) (uv)2m+1

= u2m + v2m + U 2m-Iv + ..

V ~ trai nho han .

[1+ 1/2+ (1 12)2+ .. +(1 12)"-1+(1/2)n-2](112)2m+1 < 2(1/2im+1 = (1I2im

V~ phai IOn han u2m + v2m ~(1/2)m-1 . Do do v~ phai nho han v~ trai.

V~y sinx = cosx hay x =~+ krr.

Hai 3.

-+ -+ -+ -+

-+ (p'-c')A 'C'+(p'- b')A 'B' (p -c)A 'C'+ (p - b)A 'B'

Suyra: A'M= = ' =

a' a

H6 'd~: Cho tam giac ABC, goi A', B', C' IAn hrot la trung di€m cua cac canh BC, CA, AB. Cac dirong th~ng di qua m6i dinh cuatam giac A'B'C' va ehia doi chu vi cua tam giac A'B'C' d6ng quy tai diem la dim dirong tron nQi tiBp tam giac ABC.

Th~t v~y: GQi M la di€m thuoc B'C' sao eho A'M ehia doi ehuvi tam giac A'B'C' thi MC' = p' - b' va MB'= p'_ e' voi p' = (a' + b' + e')/2.

-+ -+

= -(p- b)AB-(p -c)AC

V~y A', I, M thing hang.

Tro lai bai toan: GQi G Ia trong tam tam giac ABC. Do J trong tam h~ 4 di€m I, A, B, C nen I, G, Lthang hang.

Phep vi tv tam G ti s6 -112 bien A thanh A', B thanh B', C thanh C'. Bi~n ba dtrong thang x, y, z thanh ba dirong th~ng (theo b6 d~) d6ng quy tai I. K~t hop voi I, G, J thang hangsuy ra x, y, z, I J cung di qua mQt diem,

Hai 4.

16

-+ -+

(p -c)AC+ (p- b)AB

=

2a

M~t khac, ta e6:

-+ -+ -+

-+ -+ '-+ -+ a(AB+ AC) + (b - c) BC

2pA'[ =aA'A+ bA'B+cA'C =

2

B

c

A'

Xet each x!p x~p nhu sau :

2nk- n + 1 ... ... 2n7 .. . 4nk



2n + 1
n+l 2n 2n2 + n + 1 ... 2n2 + 2n
1 2 ... n 2nk + 1 ... 2nk +n N~u t6n tai i, j tren hang sao eho Ii - j I ~ N thi 2n2 + n - 1 ~ N.

OLYMPIC MON rOAN - 2

17

NSu a tren c9t thi 2n2 - n ~ I i - j I ~N. V~y N ~ 2n2 + n - 1. Xet N = 2n2 + n - 1.

GQi A = {I, 2, ... , n2 - n + I!. B = {3n2, 3n2 + 1, ... , 4n2}. V6i moi i . thuoc A, j thuoc B, ta co I i - j I ~3n - (n2 - n + 1) = 2n2 + n - 1. (* *)

V oi hang co chua phftn tu cua A, ta goi la hang loai 1, c9t co chua phftn tu cua A duoc goi la c9t I01;li 1. Voi hang co chua phftn tu cua B, ta goi la hang I01;li 2, voi c9t co chua phAn tu cua B, ta goi la c9t loai 2.

GQi p, q la s6 hrong hang, c9t I01;li 1 thi p. q ~ n2 - n + 1, do do

p+q~ 2jpq~~4n2-4n+4 >2n-1.

GQi r, s la s6 hrong hang, c9t loai 2 thi r. s ~ n2 + 1, do do :

r + s ~ 2.,Jr;, ~ ~4n2 + 4 >2n.

V~y p + q + r + s ~ 4n + 1, suy ra t6n tai hang ho~cc9t vira I01;li 1 I01;li 2. Til (* *) suy ra gia tri Ian nh~t cua N la : 2n2 + n - 1.

TRy'ONG THPT CHuvEN LE QUi eON. TP. eA NANG

Bili 1.

Tim t~t ca gia tri a, b, c, d, e e [0, 1] sao cho :

abc d e

A= + + + + =4.

1 + bcde 1 + cdea 1 + deab 1 + eabc 1 + abed

Bai 2.

Cho f(x)=ax3+bx2+cx+d saocho If(x)151, Vxe[-l,l] (1) Tim h~ng s6 k nho nh~t sao cho

13ax2 + 2bx + c 15k, Vx e [-1,1], Vf thea man (1).

Bai 3.

Trong m~t phang, cho tam giac d~u ABC tam O. Duong thang (d) quay quanh 0 c~t cac dirong thang BC, CA, AB IAn hrot tai M, N, P.

Cht ·nh TIl 1 khd d;:·

trng mi = --4 + --4 + --4 ong 01.

OM ON OP

Bai 4.

Chung minh voi moi gia tri nguyen cua m, t6n tai s6 nguyen n d~ n3 - lln2 - 87n + m·

chi a hSt cho 191.

Khong m~t tinh tAng quat gia S11 a ~ b ~ c ~ d ~ e (*). Khi d6:

abc d e

A~ + + + +-----

1 + abcde 1 + abcde 1 + abcde 1 + abcde 1 + abcde

a+b+c+d+e

:::-----

l+ abcde

Do a, b, c, d, e E [0, I], nen :

(1 - abc) (I - de) + (I - ab) (1 - c) + (1 -d)(1 - e) + (1 - b) (1 - a) ~ 0 (1)

. => a+b+c+d+e~4+abcde~4(1 +abcde) (2)

Do d6 A ~ 4 . N~u A = 4 thi d~ng thirc xay ra 6 (1) va (2) suy ra abcde = 0 => a = 0 do (*)

HllONG DAN GIAI

Bai 1.

de e l

c=1 d=lve=1 b = 1

Thir lai: Khi a = 0, b = c = d = e = 1 thi A = 4. V~y cac bQ gia tri cAn

tim (a, b, c, d,e) Ia cac hoan vi cua (0, 1, 1, 1, 1). '

Bai 2.

D~t A = f(-I), B = f(-1/2), C = f(1/2), D = f(l) thi

a=_~A+4B_ic+2D b=2A-~B-~C+~D

3333' 3333

A 4 4 D

o c=---B+-C--

6 3 3 6'

A 2 2 D

d=--+-B+-C--

6 3 3 6

2 A 2 4B 2

h(x)=3ax +2bx+c=--(12x -8x-l)+-(3x -x-I)

6 3

4C 2 D 2

- -(3x +x-l)+-(12x +8x-l . 36,

Til gia thi~t suy ra IAI, IBI, ICI ' IDI s 1 , do d6, n~u 'Vx E [ -1,1] thi :

Ih(x)1 s ~112x2 - 8x -11 + ~13x2 -x -11 + ~13x2 + X -11+ ~112x2 + 8x -11 Do IAI+IBI = max(IA -BI,IA+BI) nen voi 'Vx E [-1,1] ta c6

112x2 +Sx -11 +II2x2 -8x -11 =max(116xl, 124x2 - 21) ~ 22

13x2 + x-II + 13x2 - x-II = max(12xl, 16x2 - 21) ~ 4

18

B

A'

C

22 16 '

~ Ih(x)I~6+3=9, V'xe[-I,I]

VOi f(x) = 4x3 - 3x thi: V'x e [-1,1] d~t x = cost ta c6If(x)1 = Icos3tl ~ 1 va max 13ax2 + 2bx + cl = max 112x2 - 31 = 9.

[-1,1] [-t,t]

Suy ra h!ng s6 k nho nhdt thoa yeu du d~ bai la 9.

Bai 3.

GQi A', B', C' IAn luot la trung di~m BC, CA, AB thi OA' = OB' = OC' = Rl2 ..

GQi i la vecto dan vi cua (d). D~t (i,OA ') = a. , x =21t/3 thi .

A

-- -- --

(i,OB') = (i,OA ')+(OA ',OB')= a. + x + k21t,

-- -- --

(i,OC') = (i,OA ')+(OA "OC') =0. -x + k21t

T = 1 [(OA ,)4 +(OB')4 +(OC')4]

(R/2)4 OM ON OP

=~~[COS4o.+COS4(o. +x)+cos4(o. -x)]

= :4 [(l + cos 2o.i + (l + cos(2o. + 2X))2 + (1 + cos(2o. - 2X))2] 4

= R4 [3 +2(cos2o. +cos(2o. + 2x}+cos(2o. -2~))]

+ :4 [cos2 2o.+cos2(2o.+2x)+cos2(2o. -2x)] (1)

2s inx[ C9S 20. + cos(2o. + 2x) + cos(2o. - 2x)]

= [sin(2a + x) - sin(2o. - x) + sin(2o. + 3x) - sin(2a + x) + sin(2o. - x) - sin(2o. - 3x)]

= sin(2a+3x)-sin(2a -3x) = sin(2a+21t)-sin(2o. -21t) = 0 (2)

cos22a+cos2(2a+2x)+cos2(2o. -2x)

1

= 2 [3+cos4o. +cos(4a + 4x)+cos(4o. -4x)]

2sin 2x[cos4o. + cos(4o..+ 4x) + cos( 4a - 4x)]

=[sin( 40. + 41t) - sin( 4a - 41t)] = O. (3)

19

(1), (2), (3) ~ T = ~ (3 + ~) = 1 ~ khong d6i.

R 2 R

Bai 4.

(Duoc chon lam d~ thi - xem dap an d~ thi chinh thirc).

TRu'O'NG JHPI PHAN CHAu TRINH. TP. SA NANG

Bai 1.

GQi chan cac duong cao trong tam giac ABC la A', B', C'. Tinh cac goc cua tam giac A'B'C' theo cac goc A, B, C. Chung minh r~ng goc IOn nhdt cua tam giac A'B'C' it nhdt bang goc 16Q. nhdt cua.tam giac ABC. Khi nao thi cluing b~ng nhau ?

Bai 2.

Chung minh r~ng:

. 29xyz

xy + yz + zx :::;; - + _, __ .

7 7

Trong do, x, y, z la cac s5 thirc khong am thoa di~u kien

x+y+z= 1.

Bai 3.

Cho a , b , c la cac s5 nguyen dirong thoa di~u kien 111

- - - - -

abc

va d la iroc chung IOn nhdt cua chung. Chung minh r~g abed va d(b - a) nhtrng s5 chinh phirong .

Bai 4.

Chung minh rang h~ phirong trinh :

{Xy + yz + zx = 12 xyz-x-y-z =2

co mot nghiem duy nhAt trong ~p cac s5. thirc dirong. Chung minh r~g h~ co nghiem voi x, y, z thirc phan biet,

20

).

HUaNG DAN GIAI Bai 1.

Til cac tu giac n9i ti~p, ta c6 cac g6c b~ng nhau nhir hinh ben. Lai goi ten cac g6c a, '13, y nhu hinh ve. Th~ thi

A' = 2a ; B' = 213 ; C' = 2y

va A = 13 + y ; B = y + a; C = a + 13.

Suy ra d~ dang: A' = B + C - A ;

BI'-L---AL,----____;:::jo.c B' = C +A - B ; C' =A + B - C.

Bay gio, dS y rang

A' ~B' ,~B + C -A~C +A-B ~B ~A.

Do d6, n~u A ~ B ~ C thi C' ~ B' ~ A'. Cho nen

. A :$; C' ~ A :$; A + B - C ~ B ~ C : dung.

DAu dfutg thirc xay ra khi tam giac ABC d~u.

A

Bai 2.

N~ 7 h~ hi 1 9x 9xyz

eu x ~ -, t e t 1 :$; - suy ra xy :$; --.

9 7 7

H N ( ) 2' A 2 2

an mra y + Z :$; -, nen xy + xz < - < -.

997

Nhir th~ trong tnrong hop nay ta thu diroc

2 9xyz

xy + yz + zx < - + --.

7 ' 7

BA " ., di h 7 'h~}' 1 9x 0 T ' ay gro gia In x < '9' va tea - 7 > . a co

(Y+Z)2

yz s; ,

4

voi ding thirc xay ra khi va chi khi y = z. Ta cling c6 (l_X)2

yz :$; .:______::_

4

Til d6 ta chi cAn chirng minh rling

9x (l= x)? 2

(1 - - ) + x(I - x) :$; -

7 4 7

~ (7 - 9x)(1 - xi + 28x(1 - x) :$; 8

~ 9x3 + 3x2 - 5x + 1 ~ 0 ~ (x + I)(3x - Ii ~ 0,

bAt dl1.ng thirc nay hien nhien dung (vi x. khong am ), voi ding thirc xay ra

khi va chi khi x = ! .

3

21

(*)

V~y bAt d~ng thirc dll diroc clnrng minh, voi dAu d~ng thirc xay ra khi va chi khi x = y = z = !.

3

Bai 3.

GQi A ;;= ~ ,B = b ,C = ~ , th~ thi A, B, C co uoc chung Ian nhAt

d d d

la 1. Ta co _!_ - _!_ = _!_ nen AB = C(B - A).

ABC'

Ta chirng minh rkg (B - A) phai la mQt s6 chinh phirong. N~u trai lai, thi phai co mot s6 nguyen t6 p sao cho luy thira cao nhat cua p chia h~t

(B - A) la p2r+1 voi r nao do. .

Bay gio n~u pr+1 chia h~t A, thi no cling chia h~t

B = (B - A)+ A.

Suy ra p2r+2 chi a h~t AB. Ma p khong th~ chia h~t C (vi A, B, C khong co thira s6 chung ), suy ra p2r+2 chia h~t (B - A). VO li ! Do do qua lim la pr chia h~t A, va ta co k~t qua nrong tu voi B. Tir do qua lim p2r chi a h~t AB, r5i thi chia h~t (B - A). VO li ! Di~u do dll chirng minh ding (B - A) la mQt s6 chfnh phirong. Ma

ABC(B - A) = ABAB, nen ABC(B - A) phai la s6 chfnh phirong, rbi thi

ABC(B - A) = ABC B-A

ciing la s6 chfnh phuong. Nhir th~

(b - a)d = d2(B - A) va abed = d4ABC d~u la s6 chinh phirong.

Bai4.

Hien nhien (2, 2, 2) Ia mQt nghiem (trong t~p cac s6 thuc dirong). N~u ta coi z nhu dll biet, th~ thi ta co

lIz - 2 x+y=--Z2 +1

Z2 + 2z + 12

xy =

Z2 +1

Vi th~ di~u kien it co va dud~ x, y, z la cac s6 dirong la :

. (lIz - 2i > 4(z2 + 2z + I2)(z2 + 1)

(no bao dam r~ng x va y d~u thuc ) va z > 1_ (d~ x, y va z d~u dirong). 11

(*) tro thanh

22

23

4Z4 + 8z3 - 69z2 + 52z + 44 ~ 0 ~ (z - 2i(2z + 11 )(2z + 1) ~ O.

f)'~ av d h" - khi 11 1, 2 NA hi ,

leu nay udc t oa man I - - ~ z ~ - - va z = . en c 1 CO

. 2 2

nghiem duong duy nhAt la 'f = 2 (rbi til do x = 2, y = 2, han ntra cling vi h~ phuong trinh la doi xirng doi voi x, y va z).

V oi cac nghiem thuc khac ta nen xem xet tai cac gia tri thuoc

khoa , 11 d" 1

oang tu - - en - -.

2 2

V6i z= - 1 ta diroc nghiem khong phan biet (-1, -1, - 121), loai.

V'· 2 hi d 12+251 12-251 h' d~

(Yl z = - t 1 UCYC X = - y = - c ung eu

. 5' 5'

phan biet,

~ TR!IOJtG THPT HOANG HQA THAM. TP. DA NANG . Cho 3 sl, dirong a, b, c thea: _!_ + _!_ + _!_ = 3

a . b c

Chirng minh : if;! +W +W ~W +W +W.

Bai 2.

Giai M phuong trinh:

Xl + x2 - x3 - X4 < 0

(Xl +X2)(X3 +X4)-XlX2 -X3X4 <0 (Xl +X2)X3X4 -(X3 +X4)XlX2 <0 Xl >0, X2 >0, X3 >0, X4 >0

Bai3.

Tim tAt ca cac sl, t\l nhien gbm 3 chtt sl, sao cho m5i sl, la trung binh cong cua cac sl, suy til sl, do bang phep hoan vi tron cac chtt sl, cua sl, do.

Bai 4.

Tinh sl, canh cua 1 da giac d~u co 4 dinh lien ti€p A, B, C, D thoa

, 1 1 1

dang thirc : -=-+-.

AB AC AD

Hu'O'NG DAN GOO

Bail.

Ap dung b~t dfutg thirc Co-si : til gia thiet, ta c6 :

3 = .!.+l.+.!. ~ 3~ ~ abc ~ 1. abc

D~t x = t.{;., y =~, z = I{/c .

Bai toan tro thanh :

. {x > 0, y > 0, Z > 0 9 9 9 8 8 8

Val , chirng minh x + y + z ~ x + y + x .

xyz ~ 1

. Ap dung b~t dAng thirc Co-si cho 9 s6 :

x 9 + + X 9 + 1 ~ 9x 8 (1 )

y9 + +v' +1 ~9y8 (2)

z9+ +z9+1~9z8 (3)

Ta c6: x8 + y8 + z8 ~ 3~x8y8z8 ~ 3 (4)

Cong v~ theo v~ cac bdt d~ng thirc (1), (2), (3), (4) ta c6 k~t qua dn chirng minh: x" + y9 + Z9 ~ x8 + y8 + Z8 .

Bai 2.

Xet h~ bdt phirong trinh

A = -XI - x2 + X3 + x, > 0

B = XIX2 - XIX3 - XIX4 - X2X3 - X2X4 + X3X4 > 0 C = XIX2X3 + XIX2X4 - XIX3X4 - X2X3X4 > 0

D = XIX2X3X4 > 0

D~t f ( x) = (x - xl)( X - x2)( X + x3)( X + x4) . Khai trien ta c6 f{x] = X4 +AX2 +BX2+CX + D.

Do cac h~ s6 A, B, C, D > 0 nen phuong trinh f (x) = 0 khong th€

..

c6 nghiem x dirong. Vi th~ XI' x2 :$; 0 , trai gia thiet.

V~y h~ da cho la vo nghiem.

Bai3.

sf> phai tim c6 dang abc, voi a, b, c E N va I:$; a s 9, O:$; b, c s 9 .

Th ., h·t. -b bca+cab' 189 8Ib 8

eo gra t let: a c = ~ a = + 10 c

2

24

~ 7a =3b+4e ~ 7(a-b)=4(e-b)(1) Suy ra:

4(e-b) : 7 (2) ~e-b : 7 (3)

Do O:s; b, e :S; 9 nen -9:S; e - b :S; 9 (4 )

Til (3) va (4) suy ra e - b = -7,0, 7. Xet cac tnrong hop :

TH1: e-b=-7~b=e+7:S;9 ~ e = 0,1,2

~ b=7, 8,9

~ a =3,4,5

~ abc = 370, 481, 592.

TH2: e-b=O~ b=e ~a=b=e

~ abc = 111, 222, , 999.

TH3 : e - b = 7 ~ e = b + 7 :S; 9

~ b = 0, 1, 2, e = 7, 8, 9 ~ a =4,5,6 .

~ abc = 470,581,692.

V~y co tAt eel 15 s6 dn tim la : 370,481,592

111,222, , 999

470, 581,692.

Bai4.

Gift su da giac dSu nQi tiep trong dirong tron tam 0 ban kinh R. D~t a = AoB (0° < a < 1200) • Ve OH .L AB , suy ra

AB = 2HB = 2R sin a .

2

Tuong tu : AC = 2R sin a, AD = 2R sin 3a .

. 2

Thay vao gia thiet :

1 1 1

-- = -- +---,----. a sin o. .. 3a

sm- sm-

2 2

25 "

D d' . . 3a . a(. . 3a) 0

o 0: smasm--sm- sma+sm-- =

2 2 2

~ ( cos ~ - cos 5; ) - ~ ( cos ~ - cos 3;) - ~ (cos a - cos 2a) = 0 , hay

( cos 3; + cos 2a ) - ( cos a + cos 5; ) = 0 I 7a . a . a 0 cos-sm-sm-=

4 4 2

Voi di~u kien 0 < a < 1200, ta co

cos 7a = 0 :::::> 7a = 900 :::) a = 3000 447

V~y da giac co 7 canh.

TRt1CmG THPT THAI PHIEN. TP. SA NANG

Bili 1.

1) Cho 3 s6 dirong co t6ng bang 4. Chung minh r~ng t6ng cua hai s6 bAt ki trong 3 s6 do khong be han tich cua 3 s6 do.

2) Tinh S = sin39 0 + sin69 0 + sin 1830 + sin213 o.

Bai2.

. . . h lnh 1 + 3J;. 1 0

lal p uang tnn : ,-;:;-:-- - = .

4x+'\I2+x

2) GQi x , y l~n IUQi la s6 do hai goc trong cua hai da giac d~u VI va V2• Bi~t rfu1g : 5x - 7y = o. Tim s6 canh cua VI, V 2.

Bai3.

Chirng minh r~ng voi moi nguyen dirong n ta co :

1 1 1 1

2 + 3.ifi+ 4.ifj+·········+ (1 + n).{/n" <3.

Bai 4.

Cho tam giac ABC can tai A. Bi8t r~ng tnrc tam H cua tam giac n~m tren duong tron nQi ti~p cua tam giac do. Tinh co sA.

26

27

HuaNG DAN GIAI

Bili 1.

1) Gia thiet cho 3 s6 duong a, b, c va a + b + c = 4. Khong mAt tinh t6ng quat, ta clurng minh : a + b ~ abc. Til (a + b) 2 ~ 4ab suy ra (a + b + C)2 ~ 4 (a +b) c

¢:> 16 ~ 4( a + b) c

¢:> 16(a + b) ~ 4(a + bic ~ 16abc ¢:> a+ b ~ abc

Dang thirc xay ra khi a = b = 1, c = 2.

2) Ta co:

S = 2sins4 0 cos 15 0 + 2sin 198 0 cos 15 o.

= 2cos 15 0 ( sin 54 0 + sin 198 0 ) = 2 cos 150 (sins4 0 - sin 18 0) = 4 cos l S °cos 360 sinl8°

4 cos 15° cos 36° sin 18° cos 18° 2 cos 15° cos 36° sin 36°

cos 18° cos l S"

= cos 15° sin 72° = cos 150 sin 72°

S=J6+J'i.

4

Bai2.

I)DK: x z O

Phuong trinh cho : 1 + 3.Jx - 4x -.../2 + x = 0 ¢:> 3.Jx -.J2+x =4x-l

¢:>'8x - 2 = (4x -1)[3.Jx +.../2 + x J

A

¢:> (4 x-I) [3.Jx +.../2 + x _ 2J = 0 ¢:> [4 x;= 1 = ~

3vx --n-:« =2

1 * 4x -1 = 0 ¢:> x = -. 4

r » 3 r: ~2 2 .,. d h'~ 7-3J5

* Val vX +vL+X = giai ra iroc ng rem : x = .

" . 8

I. 1 ~ 'h inh cc 2 h'~ l' 17-3J5·

Ket uan: P irong tn co ng iem a: x = -', x = .

. ,: 4 8

2) GQi s6 canh cua da giac d~u DI, D21§n hrot la : n va k.

DK : n, k hl s6 nguyen dirong va 3:::; k :::; n .

T ,.:. d ~., 'n l' (n - 2 )1t

ae080 o moi goe cua rr, a x=

n

V, , n}' (k-2)1t

a eua U2 a y = k

5'x _ 7y = O<=> 5(n - 2)1t = 7(k - 2)1t

n k

<=> 5nk -10k = 7nk -14n

7n

<=> 5k + nk = 7n <=> k = --

n+5

35 <=>k=7---

n+S

Vi k la 86 nguyen dirong nen 35: (n + 5). Do d6 (n+5) phai bang : 1,5, 7 hoac 35.

Nhimg vi k ~ 3 ~ (n + 5) = 35 . V~y n = 30 va k = 6.

Bai3.

V oi moi 86 nguyen dirong k, ta e6

1 1 ~-~

~ - ~k+l = ~k(k+l)

1

1 1 1 1

Suy ra - - > = -----;=

~ ~k+l ~.~k+1.(31(k+l)2) 3(~+k).~

Val < 3(_1_ 1) . Do d6

ay (l+k).~ ~ ~k+l

1 1 1 1

2+ 3ifi + 4.ifj + + (1 + n).~

1 1 1 1

<3.(1- ifi+ ifi-ifj+ .. ···· .. - ~n+l)<3.

Bai 4.

GQi 0 la tam duong tron nQi tiep tam giac, e6 ban kinh r va K 1ft trung di~m cua Be.

Goi x la 86 do g6e BHK ~ x = 900 _ A .

. 2

28

29

).

GQi y la s6 do g6c BOK, ta c6 :

2 0 0 0 A ·0 A ° A

Y = 180 - B = 180 - (90 - - ) = 90 + - => y = 45 + - .

22. 4

Tam giac BHK cho tgx = BK va ,:1 BOK cho tgy = BK = 2tgx

2r r

A A A

~ tg(45° +-) = 2tg(90o --) = 2cotg-

. 4 2 2

1 A 1 2A A. A 2A. 2A

+tg- -tg - cos-+sm- cos --sm -

~ 4= 4~ 4 4= 4 4

1 A A A.A.A A

-tg- tg- cos--sm- sm-.cos-

4 4 4 4 4 4

. A A ( A . A)2 1 2' A A

~ sm-.cos-= cos--sm- =. - sm-.cos-

4 4 4 4 4 4

.A Al .A 2

~ sm -. cos - = '-- ~ sm - = :__

4 4 3 2 . 3

A 8 1

Til d6 ta c6 cos A = 1 - 2sin 2 2" = 1- 9 = 9 .

TRU'(rNG THPT CHuvEN NGUV@N siNH KHIEM. TiNH QUANG NAM

e Giiri phirong trinh sau tren tap sO thirc :

X4 + 2006x3 + 1 006009x2+ x - ..jr-2x-+-20-0-7 + 1004 = O.

Bai 2.

Chung to r~ng neu XI, x21a cac nghiem cua phuong trinh : x2 - 6x + 1 = 0

thi voi moi n E N, s6 x~ + x~ la mQt s6 nguyen khong chi a het cho 5.

~

Xet cac s6 thuc dirong a, b, c thoa di~u kien : <

2006ac + ab + be = 2006.

. ~

T' . , . 1.(.... hJ. , b';:' h' p' 2 2b2 3

im gra tI1 Ull n at cua leu t ire = a2 + 1 - b2 + 20062 + c2 + 1 .

+ Tnroc h~t ta chirng minh :

V6i moi n E N, s6 x~ + x~ la s6 nguyen. (*)

Menh d~ dung voi n = 0, n = 1, n = 2. Ta co

x~ + x~ = 1 +1 = 2 ;

x: + x; = 6; x~ + x; = (XI + X2)2 - 2x"x2 = 62 - 2.1 = 34. Gia su menh d~ (*) dung voi n = k - 1. V6i n = k, ta co:

k k (. )( k-I k-I) (k-2 k-2)

XI +X2 = XI +X2 XI +X2 -XIX2 XI +X2

= 6(X~-1 + X~-I)_(X~-2 + X~-2)

= 5(X~-1 + X~-I) + (X~-I + X~-I) - (X~-2 + X~-2).

(**)

Bai 4.

Cho hinh thang ABCD co day nho la AB. M<)t dirong tron qua B va C ti~p xuc voi canh AD tai E, m<)t duong tron qua A va D ti~p xuc voi canh BC tai F. Hai dirong tron nay dt nhau tai 2 di€m M va N. Chung minh r~g hai tam giac EMN va FMN co dien tich bang nhau.

Hu'O'NG DAN GIAI

Bai 1.

2007 ,

UK : X ~ --2- (*). Phuong tnnh.tu~g duang :

1 . -r----

X2(X2 + 2.x.1003 + 10032)+-(2x + 2007 -.J2x + 2007 + I) = 0 2

~ X2(X + 1003)2 +_!_(.J2x + 2007 _1)2 = 0 2

{X(X+I003)=0

~. ~ x=-1003

.J2x + 2007 -1 = 0

(thoa (*». V~y nghiem cua phuong trinh la X = -1003.

Bai 2.

Do do n~u(xk-I +Xk-I) va (Xk-2 +Xk-2) la cac s6 nguyen thi x! +Xk

, I 2 I 2· I 2

Ia m<)t s6 nguyen, til do thea nguyen tic quy nap thi x~ + x~ Ia m<)t s6 nguyen voi moi n.

+ Bay gic ta chung minh x~ + x~ khong chia h~t cho 5 bang phan

chirng. Gia su co cac s6 tu nhien n sao cho : x~ + x~ chia h~t cho5.

GQi I10 la s6 tu nhien nho nhAi rna x~" +x~o chia h~t cho 5.

30

31 "

cho 5,

Th~ k boi 110 -1 trong (**) ta diroc :

X",,-I +X",,-I - 5(X",,-2 +X",,-2)+(X",,-2 +xno-2)_(X",,-3 +X",,-3)

1.2 - I 2 I 2 I 2

Til do suy ra :

X",,-3 + X",,-3 - 5(X",,-2 + X",,-2)_[(X",,-1 + X",,-I) _(X",,-2 + X",,-2)]

I 2 - I 2 I 2 I 2

cling phai chia h~t cho 5, f)i~u nay trai voi gia thi~t no la s6 tu nhien nho

nhAt rna X~" + x~" chia h~t cho 5, Do do, gia thi~t x~ + x~ chia h~t cho 5 Ia khong tbn tai,

V~y x~ + x~ khong chia h~t cho 5 voi moi n.

Bai 3 .

. T' .. h'l.· ab be l' i a b 0 A: ~ •

ir gra t let ta suy ra ac+--+-- = . I a, ,c > nen ton tal'

2006 2006 . .

A, B, C E ( 0, 1t) sao cho A + B + C = 1t, Va vi

A B' B C C A ab be

tg - tg - + tg - tg - + tg - tg - = 1 = ac + -- +--

2. 2 2 2 ·2 2 2006 2006

A;'d A b B Cth"

nen neu at a = tg-;-- = tg-;c = tg- I ta co:

. 2 2006 2 2

2 2 3 2A. 2B 2C

P=-~A-:--- ----+ =2cos --2sm -+3cos -

tg2 _ + 1 _1 _ + 1 tg2 C + 1 2 2 2

2 tg2 B 2

2

A B 3 3' 2 G 3' 2 C 2' C A-B 3

= cos +cos + -: sm "2=- sm "2+ sm"2cos-2-+

, .2C 3 '2C 1 2A-B 3 1 3 10

s....:.3sm -+ SIn -+-cos --+ s-+ =-.

2 23 2'3 3

. Til d6 : (13 sin C _ ~ cos A - B)2 ~ 0

2 v3 2 .

<=> 2sin C cos A - B s 3sin2 C +!cos2 A - B.

2 2 2 3 2

1 A-B .

cos -' -- = 1 A = B

DAu "=" xay ra khi va chi khi. 2 <=> {. C 1

. C A-B sm-=-

. 3sm-=cos-- 2 3

·22

1 J2 t:

Suyra: c= ~;a=-;b=1003~2.

2~2 2

Yay Max P = .!.Q khi c = l~;a = J2;b = 1003J2.

. 3 2~2 2

Bai 4.

GQi 1= EFnMN; K = ADnBC; P = EFn(ADF); Q = EFn(BCE) (ADF) = (0); (BCE) = (0').

Tac6 :

P K/(O) = KF2 = KA.KD (1); P K/(O') = KE2 = KB.KC (2)

KF2 KAKD KF2 KA 2 KD . KA

Suy ra: KE2 = KB.KC <=> KE2 = KB2 (do KC = KB)

• <=> KF = KA = KD <=> {KF.KB = KA.KE (3)

KE KB KC KF.KC = KE.KD (4)

Tac6 :

EA.ED = (KE .: KA)(KD - KE)

= KE.KD - KE2 - KAKD + KA.KE FB.FC = (KF -KB)(KC -KF)

= KF .KC - KF2 - KB.KC + KB.KF Til (1), (2), (3), (4) suy ra : EAED = FB.FC

Ta lai c6 : EA.ED = EP .EF = - :J>E/(O)

FB.FC = FQ.EF = - .'1'F/(O') Do d6 : EP.EF = FQ.EF => EP = FQ. (5)

Mi:\t khac, MN la true d~ng phuong cua (0) va (0'), I E MN nen -#

.11/(0) = ~J>I1(0') <=> IF.lP == IE.lQ <=> IF.(IE + EP) = IE.(lF + FQ) <=> IF .EP = IE.FQ <=> IE = IF

Di~u d6 chirng to : SEMN = SFMN (dpcm).

TRu'6'NG THPT NGUYEN DUY HIEU, TiNH QUANG NAM

r: 1 Giai phirong trinh sau ~ren t~p sf> nguyen:

!.!.x-.J2x+1 =3y-~4y-1 +2.

5

32

OLYMPIC MON TOAN - 3

33

Bili 2.

Tim da thirc P(x) voi h~ s6 th\l'c thea man :

P(x) = ~p(X2 + 1)-7+6, Vx ~ 0, P(2000) = 2006. .

Bili 3.

Trong mat phang cho 6 diSm khac nhau sao cho, cac duong th~ng n6i tirng c~p diSm trong 6 dh~m nay khong co c~p duong thang trung nhau, song song hay vuong goc nhau. Qua m6i di~m ta ke cac duong thang vuong goc voi tftt ca nhfmg duong thang co thS dung duoc va khong qua diSm do, tim s6 giao diem nhieu nhdt cua cac duong thang vuonggoc do.

Bili 4.

Cho duong tron (0) duong kinh AB = 2R, ti~p xuc duong th~ng (d) tai A. Di~m C bdt ki tren (0), tit C dung tia vuong goc va d.t AB tai D, tren

tia nay ldy diSm E sao cho CD, DE cung chieu va BC = DE. Tit E dung cac tiep tuyen EP, EQ voi duong tron (0), voi P, Q Ia cac ti8p diem. Duong th~ng EP, EQ thea thir nr dt dirong thang (d) tai N, K. Hay tinh do dai NK thea R, khi C di dong tren (0).

Bili 1.

151 x-.J2x+I =3y-~4y-I +2 (1) 1

x ~-2 {X~O

Dieu kien : y ~ ~ <=> y ~ 1

Z x,yeZ

x,ye

(l)<=>!_!x-3Y-2=.J2}S:+1-~4Y-I (2)

5

D~t p = 151 x -3y - 2; (2) tro thanh p= .J2x + 1 ~~4y-I

<=>p+~4y-I =.J2x+I =>p+4y-.I+2p~4y-l =2x+I => p2 -2x +4y -2 = 2p~4y -I' (3)

34

Ta e64y - 1 khong i8. s6 ehinh phirong, th~t v~y gia sir 4y-l=n2, n e Z,

v~ trai ia s6le nen n = 2k +·1, khi d6

4 k 2 + 4 k + 1 :::: 4 y -1 ~ 2(k 2 + k - y) + 1 :::: 0 , vo li !

Do d6 (3) ehi dung khi p = 0, v~y ta co h~

{ll

_. x-3y-2::::0 x=5

5 ~{ :::: 3 (thea phirong trinh (1)) ..

-2x+4y-2::::0 y

Bai 2.·

Theo d~ bai ta e6

P(x):::: ~"'-p(-X-2 -+-1)---7 +6; 'ifx ~O (1)

P(2000) :::: 2006 (2)

Do d6 P(20002 + 1) :::: (P(2000) - 6t + 7:::: 20002 + 7(3) f)~t XI :::: 20002 + 1 thi til (3) ta c6 P(XI):::: XI + 6

f)~t x2 :::: x/ + 1 khi d6 til (1) ta e6

p(X12 +1) ::::(P(xl)_6)2 +7:::: XI2 +7, nen P(X2):::: x2 +6.

B~ng quy nap theo quy trlnh daehQn nhir tren ta tim diroc vo han s6

hang xpx2' .... ,xn, thea:

XI < x2 < < xn < .... voi xn :::: Xn_12 + 1 va P(xn):::: xn + 6 .-

Khi d6 da thirc Q(t):::: P(t)-(t +6) e6 vo han nghiem x., x2' .... , xn , .... suy ra Q(t):::: 0, "It

V~y da thirc dn tim la P(x):::: X + 6.

Bai 3.

Theo a~ bai ta e6 s6 duong thang xac dinh til 6 diSm eho tnroc A, B, C, 0, E, F la C~ :::: 15. Qua m6i diSm e6 5 dirong thang, do d6 c6 10 dirong thang khong di qua di~m d6. Ta xet hai di~m bAt ki, gia sir A, B : cac dirong thing vuong g6e ha til A xuong cac duongthang qua B, eAt tAt ca cac duong thing vuong goc ha til B.

TRUONG HOP 1. C6 4 duong quaB rna khong qua A. V~y. tilA ta ha duoc 4 duong thing vuong gee voi 4 duang thing d6. B6n dirong thing vuong g6e nay eAt 10 dtrong thing vuong g6e ha til B tai 4.10 :::: 40 giao diSm.

TRUONG HOP 2. H~ til A eon e6 6 duong vuong g6e nita (co 10 dirong khong qua A, tnr di 4 dirong qua B khong qua A), m6i duong nay se

cit 9 duong vuong g6c ha til B (trong d6 co ldirong song song dirong con

lai), v~y c6 them 6.9 = 54giao di~m. '

Trong s5 cac giao di~m d§ xet c6 cac giao di~m trung nhau, err m6i 3 giao di~m tao thanh mQt tam giac rna 3 duong cao cua n6h\ 3 dirong vuong g6c d§ xet, v~y tnrc tam cua cac tam giac nay diroc k~ 3 lAn, s5 cac tam giac

nay la C! = 20 .

V~y s5 giao di~m nhieu nhAt c6 th~ la 15(40 + 54) - 40 = 1370.

Bai 4.

* Tac6

B

Ep2 = EQ2 = E02 _ R 2 = ED2 + OD2 _ R 2 = Bc2 -(OC2 -OD2) = BC2 -CD2 = BD2

P Dod6EP=EQ=BD.

* DiU AK ::; KP = x .

. . ,

NK == y ; EP = EQ = BD = z.

Ta c6 (0) la dirong tron hang ti~p cua tam giac ENK dod6

C

N 1

S AENK = (p - KE)R ="2 AD.NK

1 (2R ) _ (x + z + y + EN )R

<=>-y -z - -x-z

2·2'

<=> y(2R-z) = (x+ z+ y+y+x -2x -2z-z)R <=> y = 2R.

TRU'aNGTHPT CHUYEN LE QUY SON, TiNH SiNH SINH (

.

Cho ham f: N* ~ N' \ {I} thoa mfut:

fen) + fen + 1) +120 = fen + 2).f(n + 3) voi n E N· Tinh f(2006).

'. .

O<a~b~c~d _!_+~+ d ~3 abc

2 d

-+_:_~2

b c

eChUngminh: a" +b4 +c4 _d4 ~17.

Cac s6 thirc a, b, c ?uang thoa man a 2 + b2 + c2 + 2abc = 1 . Tinh gia tr] nho nhat cua

1 1 1 (2 2 2)

T=-'-+-' -+--- a +b +c .

l-a2 I-b2 l-c2

,,~

V Cac s6 thuc a, b, c, d thoa man

BAi4.

Cho L\ABC nhon, GQi (Ao), (Bo) va (Co) Ia cac duong tron dirong kinh BC, CA, AB. Til A, B va C ta ke cac ti~p tuyen toi (Ao), (Bo) va (Co). Cac tiep tuyen nay ti~p xuc voi cac dirong tron tren tai A I, A2; B I, B2 va CI, C2. Chirng minh rAng 6 di~m AI. A2; BI, B2 va C], C2 cung nAm tren mQt duong tron (C). Tinh ban kinh cua (C) theo cac canh cua L\ABC.

Hy'O'NG DAN GIAI

Bili 1.

Ta co f(2k + 1) - f(2k - 1)

= [~(2k+ 1) + f(2k) ]-[f(2k) + f(2k-l)]

= f(2k + 2) [f(2k + 3) - f(2k+l) ]

Dodo

If (3)-f (1)1 = f (4)f( 6) .. .f (2k )If(2k + 1) -f (2k -1)1

voi k e N\k ~ 2. N~u f(3) :;t:f(l)thi f(2k + I) :;t: f(2k - 1). Do do If(3)-f(I)1 ~ 2k-1 voi k e N* ,k ~ 2. (Di~u nay khongthe xay ra.)

V~y f(3) = f(1), suy ra f(2k + 1) = f(2k - I) = a.

Tuong tu f(2k + 2) = f(2k) = b voi a, b e N*;a, b ~ 2 va Vk e N*, va til gia thi~t ta co

a+b+ 120=ab <=> (a-l)(b-I)= 121 = 112

36

37

a -1 = b -1 = 11 [b = 2

{a-l = 121

¢:> ¢:> b=12

b-l=1

b=122

{a -1 = 1

b-l=121 .

[f(2006) = 2 V~y ta c6 k~t qua sau: f(2006) = 12 .

, f(2006) = 122

Bai2.

Ap dung bAt d~ng thirc Co-si cho 4 s6 ta c6 1+3a4~4a3. 24+3b4~8b3. d4+3c4~4dc3

Cong 3 bAt d~ng thirc tren ta duoc

17+3(a4+b4+c4)+d4 ~4a3+8b3+4dc3 (1)

12d 2d d,

Bat ex. = - + - + -' J3 = - + -' Y = - thi

. abc' b c ' c

4a3 + 8b3 + 4dc3 = 4a4 (ex.-J3)+4b4 (J3-y)+4c4y

= 4a 4 ex. + 4(b 4 - a 4)J3 + 4( c 4 - b 4)y ~ 12a 4 + 8(b 4 - a 4) + 4( c 4 - b 4)

= 4(a4 + b4 +c") Til (1) va (2) ta ducc di~u phai chtrng minh .

(2)

. Bai3.

Til gia thiet thi 0 < a. b. c < 1.

Do do, d~t a = cos a. b= cos J3. c = cos y voi 0< c , J3. y < ; thi

, {cos2 ex. +cos2. J3 +cos~ y = 1- 2cosex..cosJ3.cosy

gia thiet tro thanh7t

O<ex.Ay<- ,

. '.... 2

{ex. + J3 + y = 7t

Suyra 7t (1). Khid6

0< ex..J3.y <-

2

38

).

T = _1_ + __ 1_ + _1_ + sin? a. + sin! ~ + sin? y - 3. sin! a. sin! ~ sin? y

V6i (1) thi ta luon c6 sirr' a. + sin? ~+ sin? y ~.2. va 4

_1_+_1_+_1_, ~4.

sin! a. sin! ~ sin? y

Ap dung bAt diing thirc Co-si cho 2 s5 ta c6

• 2' 9 3

sm 0.+ ~-

16sin2 a. 2

• 2 ~ 9 3

sm + ' ~-

16sin2'~ 2

• 2 9 3

sm y+ ~-

16sin2 t 2

, 7( 1 1 1)' 7

va - ---+--+-- ~-

16 sin? a. sin! ~ sin" y 4

VA T 13 . T 13 khi ay' ~ - suy ra mm = - 1

. 4 4

• • R' Jj b 1

sm a. = sin p = sm y = 2 ee .a = = c = 2 .

BAi 4.

Ke duong cao AA'. GQi H la true tam ~ABC. Ta c6 BDHA' nQi ti€p nen ta diroc

AH.AA' = AD.AB = AAI2 =AAl = k.

Xet phep nghich dao

Nl: AI 1-+ AI A21-+ A2

. A'I-+ H Tac6 5 diSm A, A I, A' , Ao, A2 cung nAm tren duong trim (AAo) duong kinh AAo . '(3)

Tit (2) va (3) thi Nl: AIA2 1-+ (AAo). Suy ra HE AIA2

(1)

A

(2)

va HAI.HA2 = HA.HA' (4)

Tuong tv goi B', C' la chan dtrong cao ke tit B, C cua MBC. Ta dircc

(5)

39

"

HCI,HC2 = HC.HC' (6)

-- ----

Do H lit tnrc tam ~ABC nen HAHA'= HB.HB'= HC.HC' . Tit (4), (5), (6)

ta c6 HAI.HA2 = HBI.HB2 = HC! .HC2. (7)

AAo, BBo, CCo la cac duong trung tnrc cua AIA2, B1B2, C1C2. K~t hQ'P voi (7) thi 6 di~m A I, A2, B I, B2, CI, C2 cung n~m tren dirong tron (C) tam G (G la trong tam ~ABC ).

GQi R la ban kinhcua (C). Do G thuoc AAo nen sir dung dinh li Stewart ta diroc

2 b2 2

GA 2 = a + +.c

I 18

mUONG THPT DAN lip DUY rAN,riNH PHU YEN

. eChO b. sf, thuc duong a b, c tho. dieu kien a + b + c ~ 1. TIm gill tri nho nhdt cua bieu tlnrc

M= 1 +_1_ .

. 1- 2 ( ab + be + ca) abc

Cho da thirc b~c 4 :

P(x) = 2006x4 + 2004x3 + 2007x2 + 2003x + 2005.

Chirng minh rkg: P(x) > 0 Vx E R.

Cho hinh thang can ABCD (AD = BC) nQi ti~p trong dirong tron tam 0 ; mQt ducrn~ thang ~ song song voi AB c~t duong tron l~n hrot tai .M, N. Chung minh rang true tam cua cac tam giac AMD, MCB, CAN thang hang.

Bili4.

Cho tu giac ABCD c6 ABc~t CD tai E, BC c~t AD tai F. Chung minh rkg cac true tam cua ban tam giac ABF, ADE, BEC, DCF thang hang.

Hu'6'NGDANGW

Bai 1.

Theo gia thiet : .

a + b + e = 1 <=> a2 + b2 + e2 + 2(ab + be + cb) = 1

<=> 1-2(ab+be+ea)=a2+b2+e2•

M duoc viet lai : M = 1 + a + b + e

a2 + b2 +e2 abc

1 1 1 1

~--:--~+- +- +_.

a2 + b2 +e2 abbe ea

Ta co: (ab + be + ea)(_l +_1 +_.1 ) ~ 9 {Theo BUT Co-si). ab be ea

1 1 1 9

~-+-. +-~---ab be ea ab+ bc+ca

Ta duoc:

1 9 127

M~ + = + + .

a2+b2+e2 ab+bc+ca a2+b2+e2 ab+bc+ca ab+bc+ca

Ap dung BUT Co-si eho ba s6 duong, ta co :

r-~-~-------

121

2 2 2 + ~ 3'3 ( . .) 2

a +b +e ·ab+be+ea a2+b2+e2 [ab-s bc+ca]

va : ~(a2 +b2 +e2)(ab+bc+ea)2 s

a2 + b2 +e2 + 2ab+ Zbc+ 2ea (a + b+e)2 1

< = =

- 3 . 3 3

Vi h~ 1 2 > 9 T ~ ,

I t e 2 2 2 + - . a cung co :

a + b + e ab + be + ea

a2 + b2 + c2 ~ ab + be + ca

<=> a2 + b2 + e2 + 2ab+ 2be + 2ea ~ 3(ab + be +ea)

<=> (a + b + C)2 ~ 3 ( ab + be + ca) <=> 1 ~ 3 ( ab + be + ea)

<=> 1 ~3<=> 7~21

ab + be + ca ab + be +ea

Ta diroc M ~ 9 + 21 = 30. DAu "=" xay ra khi a = b = e = _!_.

. . 3

40

41

Yay minM = 30 khi a = b = c = !.

. 3

BAi 2.

Truong hop 1 : N~u x ~ 0 thi p(x) > O. Truong hQ'P 2 : Xet x < O. Ta co :

x4 + x3 + x2 + X + 1 = ( X 4 + x3 + : x 2 ) + ( ~ X 2 + X + 1) + ~ X 2

~ X'( x+ ~)' +GX+l)' + ~ x'> 0

Ta vi~t lai P(x) :

P(x) = 2X4+ 2004(x4 + x3 + x2 + X + 1) + 3x2 - X + 1

= 2X4 + 3x2 + (- x) + 1 +2004(x4 + x3 +X2 + X + 1) > 0 \Ix E R.

V~y P(x) > 0 ; \Ix E R.

BAi3.

GQi HI, H2, H3 IAn hrot la. tnrc tam cua cac tam giac AMD, MBC, CAN. R5 rang ba tam giac nay d~u n9i ti~p chung duong tron ngoai ti~p hinh thang can ABeD. Theo tinh chAt true tam, ta co:

OH1 =OA+OM+OD (1)

OH2 =OM+OC+OB (2)

- ---

OH3 =OC+OA+ON (3)

Tit (1) va (2) suy ra :

Do AB II DC II MN nenCD = aAB , NM = J3AB V~ytit (4), (5) suy ra

H2HI = ( a -1) AB , H2HJ = (J3 T 1 )AB => H2HI = tH2H3

Do do H1H2'IIH1H3, suy raHi, H2, H3 thang hang (d.p.c.m).

Bai4.

Tnroc het ta chimg minh b6n dirong tron ngoai tiep b6n tam giac c6 chung nhau mot di~m P. _--_

GQi P la giao di~m cua (EBC) va (CDF) the tht CPE =1800 -CBE = 1800 -(W+BFA)

=> CPE+CPD=1800-EAD

=> EP5 + EAD = 180°.

Suy ra til giac ADPE noi ti~p, v~y duong tron (ADE) qua P.

Tuong tu, ta clnrng minh dirong tron (ABF) cling di qua P. GQi M, N, R, S la chan cac duong vuong g6c ha til P xuong AB, CD, BC; AD. Suy ra M,N;R, S thing hang (Duong thing Simson). GQi HI, H2, H3, H4 l~n hrot la true tam cac tam giac ABF, ADF, BEC va DCF. Theo tinh chdt du~ng thing SimSon, suy ra trung di~m cua PH), PH2, PH3, PH4nfun tren dirong thing Simson MNRS. Til d6, HI, H2, H3, H4 thing hang (dpcm).

TRUONG THPJ CHuvEN LlJ'gNG VAN CHANH. TiNH PHU VEN

BM 1.

Cho x, y, z la cac s6 thirc thoa man :

x + y > 0; x + z > 0; y + z > 0; xy + yz + zx > 0 .

42

Chung minh rkg: x.a2 + y.b2 + z.c2 ~ 4~xy + yz + zx.S

(a, b, e 1ft dQ dai 3 canh tam giac, S la dien tich tam giac),

Bai 2.

ABC

Cho tam giac ABC thea cotg-+2cotg--23cotg- = O.

2 2 2.

Tim GTNN cua eosC.

Bai 3 ..

Cho tam thirc f(x) = x2 - 2mx + 1, m e (~ ;:). Giai phuong trinh

[f(X)]2 - 2mf(x) + 1 = x. (1)

Bai 4.

Cho duong tron tam 0 ban kinh R va mQt di~m A tren dirong tron d6, tren ti~p tuyen voi duong tron tai A lAy di~m M sao cho MA = R. Qua M d\II1g mQt cat tuyen thay d6i c~t (0) tai B va C (B nk gitra M va C). Tim vi tri B, C sao eho SASe dat gia tri 100 nhAt.

Hl16'NG DANGIAI

Bai 1.

Ta e6 : x.a' + y.b2 + z.e2 = x(a2 - b2 - e2) + (x + y)b2 + (x + z) c2

= - 2xbe. eosA + (x + y)b2 + (x + z)e2

~ -2bcx.cosA + 2~( x + y)( x + z ).bc (Co-si)

hay x.a2 + y.b2 + z.e2 ~ 2bc( ~( x + y )(x + z) - x.eosA}.

Ta lai e6:

( x - ~r-( x-+-y-)-( x-+-z-).eosA r ~ 0

::::) x2 +( X + y)( x + z)cos2 A - 2x~(x + y)( x + z).cosA ~ 0 Til d6: .

(x + y)(x +z)+ X2COS2 A- 2x~(x+y)(x +z).cosA -( xy +yz+zx)sin2 A~ 0

~ (~(x+y)(x +z) -x.cosA r ~ (xy+ yz+zxj.sin ' A

~ ~(x+y)(x+z) -x.cosA ~ ~(xy+ yz+zx).sinA

Do d6: x.a' + y.b2 + z.e2 ~ 2bc~xy+yz+zx.sinA. Ma 2be.sinA = 4S VfJ.y x.ai+ y-.b2 + z.e2 ~ 4~xy + yz+ zx.S (dpem).

44

Bai2.

Til gia thi~t ta co

ABC

cotg-+2cotg--23cotg- = 0 (1)

2 2 2

Ta dua (1) v€ dang

B C C A A C

x(cotg-+cotg-)+ y(cotg-+cotg-)+ z(cotg-+cotg-)= O.

2 2 2 2 2 2

D6ng nhat cac h~ s6 ta.ducc x = - II,y = 12, z = 13. Do do

ll(cotg B +cot g C)+12(cotg C +cot g A) = 13(cotgA +cot g B)

2 2 2 2 2 2

. (B C) . (C A) . (A B)

sm 2+2 sm 2+2 sm 2+2

~ 11 . B . C +12 . C . A = 13 . A . B

sm-sm- sm-sm- sm-sm-

2 2 2 2 2 2

11 A. A l' 2 B. B 13 C. C

~ cos-sm~+ cos-sm-= cos-sm-

2 2 2 2 2 2

=> II.sinA + I2.sinB ::;: 13. sinC

=> lla + I2b = 13c

=> (l Ia + I2bi = I69c2

=> I2Ia2 + I44b2 + 2.I32.ab == 169(a2 + b2 - 2abcosC)

=> 2ab(I32 + 169 cosC) = 48a2 + 2Sb2 ~ 2.20 Jj .ab

C..." 20J1-132

~cos ~ .

169

DI. b~ , khi hi khi 4 {;;3 Sb' VA . C 20J1-132

au ang xay ra I va c 1 1 " ja = . ay mmcos . = .

. 169

Bai 3.

(1) <=> (x2-2mx+ Ii-2m(x2-2mx+I)+ l-x=O

<=> 4(x2 + x)m2 - 2(x2 + 1)(2x + l)m + (x2 + Ii + I - x = 0 (2) Xet v~ trai cua (2) 1ft tam thirc b~c hai theo m, x 1ft tham s6, ta co :

L\'=(x2 +1)2 (2x+l)2 _4{X2 +X)[{X2 +It +l+ x ]

= X4 + 4x2 + 1 + 4x3 - 2X2 - 4x

=(X2 +2x-If.

N h'A , (2) 1). x2 -x'+1 x2 +x+2 T .1. 1 ..

g iem cua a : ffi, = . ; m2 ;::: • a viet ai :

2x 2(x+l)

45

4(X2 +x)[m- X2 -X+l].[m_ X2 +X+2]=0

2x . 2(x+I)'

~ [X2 _ (2m + l)x + 1].[x2 - [X2 - (2m + I)x + 2 - 2m] = 0

[X2-(2m+I)X+I=0 (3)

~ x2-(2m-l)x+2-2m=0 (4)

Taco: ~J =(2m+I)2 -4=4(m-_!_)(m+~) va

- 2 2

. . 1+2J2 1-2J2

~4 =(2m+I)2 -4(2-2m)=4(m+ )(m+ )

2 2

V ' . (1 4) {~> 0 d 'd' (4) A h' A '(3)' 2 h' "

01 m e -; - ~ , 0 0 vo ng rem va co ng rem

2 5 .6.4<0

2m+l-J(2m+I)2 -4 2m+I+~(2m+l)2 -4

XI = 2 ' Xz = 2 .

V~y phuong trinh (I) co 2 nghiem XI, X2·

GQi I la trung di~m cua BC, dung AH 1. BC (H e BC). Gia sir goc HMA = a. Qua 0 ke OP .L AH, suy ra L OAP= a. Taco:

1 .

SABC = - AH. BC = AH. IC

2

= R sina.../OC2 -Oe = Rsin a.JR 2 _ 012

= Rsina.JR2 _HP2 Ta co HP = AH - AP = R(sina - cosec). Suy ra

Bili 4.

M

A

SABC = Rsino JR 2 - R 2 (sin a - cosa)2 = R 2 sin a.../2 sin ucoso = R2J2.JsinJ u.cosu .

Taco:

sirr'u.cosn ~ .Jsin' a.cos'a ~ 27{ Sin; a )3 cos'n s .

Hu'ONG DAN GIAI

s . 27(Sin2 a +eos2aJ' = ~27. 1 = 3J3 (Ap dung ce-s cho 4 sA)

4· ~ 16

. Do d6 S ABC =R 2..fi..J sin 3 c.cosu s R 2 ..fi..~ 3J3 = JJjf R 2 •

16 . 8

V~y Max SABC = 1s R2 DAn '':'' xay ra khi va chi khi sin ' a 2 t: -- = cos a<=> tg2a = 3 <=> tga = ,,3

3

(a luon nhon) hay a = 60°.

V~y cac di~m B, C cAn tim lit giao di~m cua cattuyen M, hQ'P MA m9t goc 60° voi (0).

o

, It. ., U ..

TRUONG THPT CHUYEN HUNGY_O'NG, UNH GIA LAI

Bai 1.

Tim x n~u [~]+[~]= 5 .

. ~) .

VCho cac s5 thuc dtrong a, b ,c. Chimg minh r~ng :

(2a +b+e)2 (2b+e+a)2 (2e+a+b)2 8

2a2 +(b+e)2 + 2b2 +(e+a)2 + 2e2 +(a+ b)2:$; .

B8i 3 ..

Giai phuong trinh x3 - 3x = ~ x + 2 .

Bai 4.

Cho tam giac ABC can. Duong tron n9i ti~p tam giae ti~p' xuc voi AB tai T. CT cit duong tron tai K. Gia sir K lit trung di€m CT va

CT = 6Ji. Hay tinh d9 did cac canh cuatam giac ABC.

Bail.

Ta xet cac tnrong hQ'P:

, 3' [3] '3

a) Neu x < 0 thl - <0 => - :$; -:$;0.

x x x

·46

47

Tuong tu ta co [:] ::; O. Vi v~y PT va nghiem,

b) Neu x> 0 thi ~ < : ~ [~]::; [:].Til tinh chAt nay va dinh nghia phdn nguyen, ta co ba tnrong hop:

* [~l= 0 va [:] =5

Til [~] ~O suy ra 0,,; ~ < lsuy ra x ;> 3

Vi [~] =5 suy ra 5 ::; 4 < 6 hay la 2 < x s . 4 .

x x 3 5

Cac khoang nay khong co x nao thea man.

*[~]=1 va [:]= 4.

Til [~]=1 suy ra 1 s ~ <2 suy ra l < x ::; 3.

x x 2

Vi [:] =4 suy ra : < x s 1, cling cho ta di~u va u.

* [~]= 2 va[:]~ 3 .

.. 4 Ta diroc 1 < x ::; -. 3

4 V~y dap sf>: 1 < x ::;'"3 .

Hai 2.

Nhan xet r~ng (1) dung voi a , b , c thi cling dung voi ka , kb , kc. Vi

v~y ta co quyen gia sir : a + b + c = 3.

T '(1) (a+3)2 (b+3)2 (C+3)2 8

a co ~ + _ _;____:-- + ::;

2a2 +(3-a)2 2b2 +(3-b)2 2c2 +(3-C)2

Ta xet (X+3)2 = 1. x2 +6x+9 = 1. ( 1 + 8x+6 )

. 2X2+(3-x)2 3x2-2x+3 3 x2-2x+3

1 8x+6 4 4

= - ( 1 + ) ::; -3 + -3 x. I'

3 . (x-l)2+2

Vi v~y VT ~ 4 + ~ (a + b + c) = 8.

Ddu = xay ra khi va chi khi a = b = c. (DPCM)

Bili 3.

Tnroc h~t nh~n xet r~ng n~u phuong trinh co nghiem x thiphai co

-2~x~2. '

Til do ta d~t x = 2cost voi 0 ~ t ~ 1t . Phuong trinh dacho tra thanh

Scos ' t - 6cos t = ~2cos t + 2 hay 2cos3t = 2cos'!_.

2

T' d ' , h· A 0 41t 41t D d 'A hi A , h

ir 0 ta co ng rem t = , t = 7' t = 5. 0 0 t~p ng tern cua p irong

trinh da cho Ia T = { 2, 2 cos ~1t ,2 cos ~1t } .

Bili 4.

GQi K la trung di~m cua CT va L la ti~p di~m cua dirong tron voi canh BC. Th~ thi : CK = ~ CT (*)

Khong tinh t6ng quat ta xet hai tnrong hop:

* TH1: AB = AC hay b = c. Suy ra L trung diem cua BC CL 2 = CK.CT = _!_CT2

2

Tire Ia a2/4 = 36 hay a = 12 (1)

Ap dung dinh li eosin trong tam giac BCT, voi P = ABC :

CT 2 = BT 2 + Be2 - 2BT.BC.cos P

~ 72 = a2 14 + a 2 - 144. cosfl ~ cosf = ~ (2) (do (1)) 4

M~t khac, ap dung dinh Ii eosin trong tam giac ABC, co

b2 = C 2 + a 2 - Zca.cosf ~ cosf = al2b (3)

Til (1), (2) va (3), ta co: (a, b, c) = (8,8, 12) .

... TH2: AC = BC hay a = b. Th~ thi T Ia trung di~m cua AB. Ap dung dinh li tiSp· tuyen : CL 2 = CK.CT = ~ CT 2 (do (*) ) hay

(a - c/2) 2 == 36.

Suy ra a = 6 + c/2. (4) Ap dung dinh Ii Py-ta-go cho tam giac BCT : a 2 = C 214 + 72 (5)

Til (4) va (5) : c = 6. V~y : (a,b,c) = (9,9,6).

48

OLYMPIC MtiN TOAN - 4

49

TRUONG THPT CHUYEN KON TUM. tiNH KON TUM

8",

.

Giai phirong trinh

(x+l)~x2-2x+3 =x2 +1.

(1)

Bai 2.

Cho f'(x) = lax2 +bx+cl~I-x2 s 1 voi moi x thuocdoan [-1; 1]. Chung minh rang :

a) I a I s 4.'

b) I ax2 + bx + c I s 3

Bai3.

Cho tam giac ABC,c6 cac canh BC = a, AC = b, AB = c va rna, ms,

, a, ,

me lao IUQ1: la dQ dai cua cac dirong trung tuyen xuat phat tir cac dinh A, B.

C. Chung minh r~g : Tam giac ABC dell khi va chi khi

b I 2 .2 2

a+ +c=2"ma +mb+mc .

Bai 4.

Cho t~p hop D = { 1,2, 3, 4, 5, 6, 7, 8,9, 10, 11, 12 }. Tim ~6 t~p con D sao cho phuong trinh x + y = 13 vo nghiem tren m6i t~p con d6.

Bail.

T~p xac dinh cua phirong trinh la 'PlJ = R I>~t ~X2 -2x+3 = t, voi t ~ J2 .

Khi d6 phirong trinh (1) tro thanh (x + I)t = x2 + 1 (2). (2)<=> x2 - 2x + 3 - (x + l)t + 2(x - 1) = 0

<=>t2_(x+ l)t+2(x-I)=O <=> [t = 2

t = x-I

* Voi t = 2, ta c6 ~X2 - 2x + 3 = 2 <=> x2 - 2x + 3 = 4

<=> x2 _ 2x _ 1 = 0 <=> .[x =;1 - J2 .

x =I+J2

a) Til gia thiet ta c6" :

f( ~) =~ 3;+ ~b+C "I~ 13a+2b-J3+4cl"S. f(O) = 1 c 1 s 1,

f,(- /3)=.!. 3a _ /3 b + c :5 1 => 13a - 2b/3 + 4cl :5 8.

, 2 2 4 2

va

16al = 13a+3a+ 2b/3 -2b/3 +4c+4c- 8cl

I

:5 13a+2b/3 +4cl+13a'-2b/3 +4cl+18cl :5 8+8+8=24

Suy ra I a 1 :s 4.

b) • Xet x E ( ~ ; 1 J. ta co :

o < 2/3 x - 3 s 2/3 - 3 => t 2/3 x - 31 :sl 2/3 - 31 0> 2/3 -Ax ~ 2/3 - 4 => 1 2./3 x - 4x 1 :s 1 2/3 - 41 va 2/3I'ax2+bx+cl =1 2Jjax2 + 2J3bx + 2J3cl = 1 (3a + 2J3 b + 4c)x + ax(2J3 x -3) + c(2J3 - 4x~ :s 13a + 2/3 b + 4c)lIxl + lallxl2J3 x -31 + Ic1l2J3 -4xl :s 8 + 41 2J3 ~3 1 + 1 2J3 -3 1 = 6J3

Suy ra 1 ax2 + bx + C 1 :s 3.

• Xet x E [-I; _ ~). ta c6 :

-2J3 +3:S 2J3x+3<0=>12/3x+31 :s1-2J3 +31,

-4 + 2/3 s 4x + 2J3 < 0 => 14x + 2J3 I s 1-4 + 2J3 1

va 2J3lax2 + bx + c 1 = 1 2J3ax2 + 2/3 bx + 2/3 c 1 = 1- 2J3 ax2 - 2J3 bx - c 1

, = t (3a - 2J3 b + 4c)x - ax(2J3 x + 3) - c(2J3 + 4x~

*Vcrit=x-l,tac6~x2-2x+3 =x-l

{X -1 ~ 0 {x~ 1

<=> <=> , <=> X E ¢J

x2 -2x+3=(x-l)2 3 =1

V~y t~p hop nghiern cua phirong trinh la { 1- Ji. 1 + -fi} .

Bai2.

-

51

$1 3a- 2vS b + 4c11 xl + I ~ Ixll 2vS x + 31 + I d .I 2..fj + 4xl $ 8 + 41- 2vS + 31 + 1-4 + 2J3 I = 6Ji

Suy ra 1 ax2 + bx + c I $ 3.

• Xet x E [_ .n, J3J, tac6: 2 ' 2

'I 1 vS 2 3 2 1 P2· 1. 1

x <-=>x<-=>l-x>-=> l=-x ~->-.

- 2 -2· -4 . 2 3

Suy ra I ax2 + bx + c I s ! = 3

3

V~yl ax2 +bx + c] $ 3, 'V x E [-1; 1].

Bai 3.

• MBCd~u=> a+b+c=2~m;+m~+m~.

Vl A ABC d:l.,. b' . avS

1 u.n. eu nen a = = c va rna = mb = me = -.-.

2

Khi d6 a + b + c = 3a, va

r---~--

3a2 382 3a2 2~m:+m~+m~ = 2 -+-+- =3a.

444 '

V~y a+b+c=2Jm; +m~ +m~ .

• a+b+c=2~m; +m~ +m~ => MBC d~u. Tac6 :

2_ 2(b2+ c2)_a2 2_ 2(c2+ a,2)_b2. 2_ 2(a2+ b2)_C2

rna - 4·' mb - 4 ' me - 4

Dodo

3 /2. 2 ·2 2"a=V2(mb +mC>-ma

3 / 2 2 2 2 b=v2(mc +ma)-mb

~ c=~2(m; +m~)c-m;

suy ra

~(a+b+C)=~2(m~ +m~)-m; +~2(m~ +m;)-m~ +~2(m; +m~)-m~ .- 2

52 .

.'.,

Ap d\lIlg bAt dfu1g thue Bu-nhi-a-cep-xki, ta diroc:

~2(m~ +m~)-m! +~2(m~ +m;)-m~ +~2(m; +m~)-m~ ~Ji~3(m~ +m;+m~) hay:

~(a+b+e) s Ji~3(m~ +m;+m~) ~ a+b+e=2~m; +m~ +m~ . 2

DAu dfu1g thue xay ra khi:

~2(m~ +m~)-m; =~""'2-(m-~ -+-m-;-)---m-~ =~2(m: +m~)-m~ ¢:) m; ::: m~ = m~ => rna = mb = me hay 8ABC d~u. *V~y8ABCd~u~ a+b+e=2~m;+m~+m~.

~ _',

BM 4.

Ta eo 1 + 12 = 2 + 11 = 3 + 10 = 4 + 9'= 5 + 8 = 6 + 7. D~t: A, = {A cD 11 E A va 12E A}

A2 = { A c D I 2 E A va 11 E A}

A3 = {A c DI3 E A va 10E A}

A4 = { A c D 14 E A va9 E A}

As = { A c D I 5 E A va 8 E A}

A6 = { A c D I 6 E A va 7 E A}

GQi B la t~p hop tAt ca cac t~p concua D. GQi C la t~p hop tAt ca cac t~p eon cua D sao eho moi t~p eon Ay khong clnra tron bAt ki bl) nao trong 6 be> sf, (1; 12), (2; 11), (3; 10), (4; 9), (5; 8), (6; 7).

Khi do:

C = B \ (AtuA2UA3UA4UAsuA6)

=> I C I = I B I \ I AtuA2UA3UA4UAsuA61

(1)

Tir each d~t cac Ai (i = 1,2, 3, 4, 5, 6) va C, ta co : Neu A E C thi ' phuong trinh x + y = 13 vo nghiem, vi th~ I cI chinh lit sf, cac t~p hopcon cua D cAn tim. Ta eo :

6 I>

UAn =:LIAnl- I, IAJ)Ajl+ :L IAJ1AjnAkl-

n='1 n=1 Isi<j$6 ISi<j<ks6

:L IAJ1Aj nAk nAIl +

ISi<j<k<I$6

+ :L jAi nAj nAk nAI nAml-IAI nA2 nA4 nA; nA61'

ISi<j<k<l<m$6

Vi cac Ai (i = 1, 2, 3, 4, 5, 6) co vai tro binh d~ng nhau nen:

53

6

+ IIAnl =6IAII; I IAJ~Ajl=c~ IAII1A21 = 151AII1A21;

n=1 ISi<jS6

L IAi I1Aj I1Akl=c~ IAII1A2I1A31=20IAII1A2I1A31 (3)

ISi<j<ks6

I IAi I1Aj I1Ak I1AII =c~ IAII1 A2I1A3I1A41=15IAII1A2I1A)I1A41

ISi<j<k<IS6

I I IAi I1Aj I1Ak I1All1Arnl = 61AII1A211A311A411Asi

ISi<j<k<l<mS6

Taco:

+1 BI =213=4096.(4),

,+ Vi AII1A2I1A3I1A4I1Asl1~ = {D} nen

AII1A2riA311A411AsI1A61 = 1. (5)

+ M6i phAn tu cua AI co dang {I; I2} vYI, voi YI hi mot t~p con bAtkicUa D\{1; 12}. Vith~IAllchinhb~ngs6~pconD\{I; 12},til'do:

I D\{I; I2} I = 10 ~ I Ad = 210. (6)

+ M6i phAn tu cua I AI 11 A21 co dang {I; 12; 2; II} vY2 voi Y2 la mQt ~p con bAt ki cua D\{I; 2; 11; I2}. Vi th~ I All chinh b~gs6 ~p con D\{I; 2; 11; 12l. Til'do:

ID\{1;2;11;12}1 =8~IAII1A21 =28.(7) Tuong t1,1' ta diroc:

+ I AII1A211A31 = 26, (8)

+ I AII1A211A311A41 = 24, (9)

+ I AII1A211A311A411Asi = 22. (10)

Til' (2), (3), (5), (6), (7), (8), (9), (10) ta co:

I AlvA2VA3VA4VAsvA61

= 6.2IO~ 15.28 + 20.26 - 15.24 + 6.22 - 1 = 3367. (11)

Til' (1) va (11), ta co s6 t~p con cua D dn tim la:

4096 - 3367 == 729.

JRU'CmG THP'I cHuyEN NGuyeN OUr TiNH pAc LAc

fuW "

~ Tim da thirc P(x) thoa man: '

P(P(x» + 1 = [p2(X) + 2P(x) + (x2 +3x + 1/]2.

Bai 2.

tho n di~m A J, A2, ... , An tren m~t ph~g, khong e6 ba di~m nao th~g hang, khong e6 4 di~m nao tao thanh hinh binh hanh, GQi I)., h, ... , 1m lit tAt ea cac trung di~m tao thanh til cac doan th~g e6 dAu milt lit hai di~m Ai, Aj nao d6 (1 ~ i, j ~ n). GQi M lit tAng dQ dai moi thing co dAu milt lit hai di~m Ai, Aj bAt ki (1 ~ i, j ~ n ). GQi N lit tAng dQ dai moi thang e6 dAu milt lit hai di~m li,Ij bAt ki (1 s i, j s m). Chirng minh rfulg:

N~ n2-3n+2.M. 4·

~1 .

Cho tam giac ABC' nhon nQi ti~p dirong tron tam O. M, N, PIa trung diem cac eung nho BC, CA, AB. Tren canh AB ta lAy hai di~m A), B2 ; tren canh BC lAy hai di~m BI. C2; tren canh CA lAy hai di~m C), A2 sao eho:

AAJ = AA2 = BBt = BB2 = CCI = Ce2.

LAy A3 lit giao PAt voi NA2 ; B3 lit giao MBt voi PB2 ; C3 lagiao fnQai MC2. Chimg Minh rins.. AA" BBj, CC, dbng quy.

D- Cho a, b, e lit cac s5 thuckhong amthoa man: a + b + e = 3. Tim gia tri Ian nhAt cua : A = 9ab + 10ae + 22be.

Hu'ONG DANGIAX

Rai 1.

P(P(x» + 1 = [p2(X) + 2P(x) + (x2 + 3x + 1/]2 (l) Ta e6 Ptx) = 0 khong tho! man, v~y P(x) e6 dang :

. P(x) =anxn + an_1xn-1 + ... + a1x + ao; (an ¢ 0)

Khi khai trien hai v~ cua (1) thi s5 hang Ian nhAt eu'a P(P(x» +1 lit:

a (a xn)n ='an+1xn2

n n n

Con s5 hang Ian nhAt cua [p2(X) + 2P(x) + (x2 + 3x + 1 )~]2 lit:

54

55 "

((anxnrf =a!x"n; n>2

((anx2t +x4f =(a~+I)2x8;n=2 ,,8. n < 2

,

V~y n~u n ~ 2 thi xn2 = x8 => n2 = 8 (ve li) Do d6 n > 2 va an+1xn2 = a4x4n => n = 4'a =1

,. n. n 'n'

V~y P(x) c6 dang : P(x) = X4 + a.x' + a2x2 + a1x + ao'

f)~t G(x) = P(x)-(x2 +3x+l)2 +1, ta c6:

(1) <=> P(P(x» + 1 = [p2 (x)+ 3P(x) + 1- P(x) + (x2 + 3x + 1)2 -1 T

<=> P(P(x»+1 =[p2(x)+3P(X)+I-G(X)T

<=> P(P(x» ~[P2(x)+3P(x) + IT + 1 = G(x)[ G(X)_2(p2(X) + 3P(x)+ 1) ] <=> G(P(x» = G(x)[ G(x) - 2(p2(X) + 3P(x) + 1) ]

N~u G(x):I: 0, d~t degG(x) = k thi k ~ 3 nen ta co:

{deg G(P(x» = 4k

degG(x)[ G(x)- 2(P2(x)+3P(x)+ 1) ] = k +8

=> 4k = k+8=> k =! (ve Ii).

3

V~y G(x) = 0 hay P(x) = (x2 + 3x + 1)2 -1= x(x + l)(x + 2)(x + 3).

Bai2.

V oi m6i tam giac ABC co trung di~m cac canh AB, Be, CA la M,

N, P, ta co ding thirc : .

MN+NP+PM:= !(AB+BC+ CA). (*)

2

B

P

N

A

c

M

V6i m6i tt'r giac ABCD (co thS t\)' cit hoac khong h~i) co trung diSm cac doan AB, CD, BC, DA, BD, AC Ia M, N, P, Q, R, S ta co bAt d~ng thirc

MN + PQ + RS :s ~ (AB + CD + BC + AD + BD + AC). (**)

Til cac di~m AI, A2, ... , An, ta I~p moi tam giac, nr giac co cac dinh thuoc cac diSm nay, r6i I~p moi d~ng thirc (*), BDT (**) nrong irng voi m6i tam giac, tt'r giac tren va cong v~ thea v~ tAt ca ta diroc mot bAt d~ng thirc (***).

Vi m6i doan Ijlj chi thuoc dung met tam giac hoac mot tir giac tren nen no co m~t dung mQt IAn ben v~ trai cua BDT (***), V~y VT(***) = N.

M~· d AA' ~ d" 2 .,' (n-2)(n-3) , "

01 oan j j co mat trong ung Il- tam giac va 2 tu giac

nen M s6 cua no khi khai triSn VP(***) la

_!_(n-2+ (n-2)(n-3)) = n2-3n+2.

2 2 4

V~yVP(***)= n2-!n+2 .M.

Do do N< n 2 - 3n + 2 .M - 4

Bai 3.

Xet dirong tron tam I nQi ti~p tam giac ABC voi cac ti~p diem Mj, Ni, PI cua cac canh BC, CA, AB. GQi K la P tam vi tu ti s6 dirong cua hai dirong tron (I) va (0). Vi I MI 110M (cung vuong goc BC) nen ta co phep vi 1\)':

R

V Z :'(1) -+ (0)

MI-+M

NI-+N

Pl-+P

Vi AA I = AA2 va AP I = AN I nen ta co phep vi nr:

AAI

V:PI : PI-+AI

NI-+A2

GQi phep vi tu tam H Ia tich hai phep vi tu tren thi H, A, K thang hang va H, P, Al thang hang, H, N, Ai thang hang. V~y H trung A3, hay AA3 di qua K.

56

57 "

Tuong tu, BB3, CC3 di qua K nen AA3, BB3, CC3 d6ng quy.

Chit y : Thi sinh co th~ dung dinh li Me-ne-la-uyt - Xe-va thay cho

tich hai phep vi tu. .

Bai4.

Ta co : a + b + c = 3,

A = 9ab + IOac + 22bc

= 9ab + IO(a + b)c + 12bc

= 9ab + IO(3-(a + b»(a + b) + 12b(3-(a + b) = -10(a + b)2 + 30(a + b) - 12p2 +36b - 3ab

Xet f(t) = - t2 + 3t voi 0 ~ t ~ 3 taco: maxf(t) = :' dat tai t = ~ .

V~y A = 1 Of (a + b) + 12f(b) - 3ab:S 22maxf(t) = 9;.

Do do GTLN cua A hi 99, xay ra khi a = 0, b = c = 2.

2 2

G

TRu'O'NG THPT CHUYEN THANG LONG. f)A LAT

Bail.

G9i K la tam dirong tron nQi ti~p tam giac ABC, BI, CI thee thir tl)' 1ft trung di~m cua cac canh AC, AB. Duong thang CIK c~t dirong thang AC tai B2, duong thang B I K dt duong th~ng AB tai C2 sao cho dien tich tam giac ABC bang dien tich tam giac AB2C2. Tinh goc CAB.

Baii.

Chung minh r~ng trong tam giac ABC bAt ki ta co :

. A . B . B . C . C. A 5 r

sm -sm - + sm -sm - + sm -sm - ~ -+-

2 2 2 2 2 2 8 4R'

trong do R, r thee thir nr 1ft ban kinh dirong tron ngoai tiep, nQi ti~p tam giac ABC.

Bai 3.

Cho n s6 thirc XI, X2, X3, ... , Xn thea man diSu kien:

X2+X2+X2+"'+X2 =1

I 2 3 n

58

"

®iAi M phuong trinh :

{X2(Y+Z)2 ~(3X2 +X+I)y2z2 y2(Z+X)2 = (4y2 +y+1)Z2X2. Z2(Y+X)2 =(5z2+z+1)X2y2

Bai 1.

f)~t BC = a, CA = b, AB = c, AB2 = X, AC2.= Y . Phan gh'lcBK e!t canh AC tai D. Ta co :

KB .. c a a+e

-=-=-=-->1.

KD AD CD b

Suy ra ; AD = ~ va D nAm gitra A va Bi a+e

Apd\ll1g dinh If Menelaus vao tam giac ABD va dirong thang B2 K C I ta co :

, B2D. CIA. KB=1 B2A CIB KD

bc

x---

a+e'~=1

x b

bc

~ x = (1)

a+c-b

Suy ra:

be

Tirong tu ta co : y = (2)

a+b-c

Til gia thiet : di~n tich tam giac ABC = dien tich tam giac.AB2C2,SUY ra : xy = be (3)

Til (1), (2), (3) ta diroc :

Taco:

59 ).

tg A (sinB+ sin C) = cosB +cos C 2

tg B (sinC+ sinA) = cos Cv.cos.A

2 '. '

tg C (sinA+ sinB) = cosA+cosB 2

Va

4R·A.B.C sm-sm-sm-

r = 2 2 2 = cos A + cos B + cos C -1

R R

Ap dung bdt diing tlnrc Co-si :

A . B B. A' 2 A. B B. A

tg-sm +tg-sm ~ tg-sm tg-sm

2 2 . 2 2

. A . B 1 ( A. B . B . A) <=> sm-sm-~- tg-sm +tg-sm

2 2 4 2 2

Tirong t1,r ta co:

. B . C 1 ( B. C C. B)

sm-sm-~- tg-sm +tg-sm

2- 2 4 2"· 2

. C . A 1 ( C. A A. c)

sm-sm-~- tg-sm +tg-sm

2 2 4" 2 2

CQng cac bdt diing thirc nay lai ta diroc:

.A.B .B.C.C.A sm _:_sm - + sm - sm - + sm -sm-

22 22,22

~ '!'[tg A (sinB+ sin Cj+tg B [sin Cs-sin Aj+ tg C (sin A + sin B)]

4 2 2 2

cos A + cos B + cos C cos A + cos B + cos C cos A+ cos B + cos C -1 1

= = + +-

2 4 4 4

.3·

Ta co: cosA + cosB + cos C .~ - ,suy ra:

2

.A.B .B.C .C.A 5 r

sm-sm-+sm-sm-+sm-sm-~-+-.

2 2 2 2 2 2 8 4R

Bai 3.

Bdt ding thirc da'cho tuang duong'voi:

[ 1 (I

n 1---+--

. I+x~ I+x;

[XI X2 . Xn ]2 n (I)

--+ + ... + . <-

I+X2 I+X2 +X2 I+X2 +X2 +···+x2 2

I I 2 I 2 n

Theo bdt d~ng thirc Bu-nhia-a-cop-ski ta c6:

V~tnii(I)~n.[(~J.2 +( ~2 2J2 + ... +( 2· ~n 2J2] (2)

l r x: l+xl +X2 l+xl +X2 +···+X"

Suy ra v~ phai cua ( 2) be hon:

(ddu < nghiem ngat vi c6 it nhdt mot x; > 0, i = 1,2, ... , n ). Bdt d~ng tlnrc dB. cho dircc clnrng minh.

Bai4.

I. Truong hop c6 mot trong ba ~n x, y, z bang 0. I) X = ° : Ta c6 h~ phuong trinh :

{o = y2z2

y2z2 =: ° Z2y2 = °

suy ra : y = 0, Z = t E R hay z = 0, y = t E R , trong tnrong nay h~ c6 nghiem ( x,y, z) E {( O,O,t), ( O,t,O), t E R }.

2) y = ° : Tuong nr ta c6 nghiem :

( x,y, z) E {( t, 0, 0 ), ( 0,0, t ), t E R }. 3) Z = ° : Tuong tu ta c6 nghiern .:

( x,y, z) E {( t, 0, ° ), ( 0, t, ° ), t E R }.

,60

61

II. Tnrong hop xyz :# 0.

Chia m6i phirong trinh trong he cho (xyzi ta co:

(;<)' =3< + :'

(.!. + 1.)2 = 4 +.!. +_1

x Z Y y2

(.!. + .!.)2 = 5 + 1. + _!_

Y x Z Z2

D~.t h 1 1 1 , h"

~t an P u Xo = -, Yo = -, Zo = - ta co ~ sau:

x y Z

{.(ZO+Yo)2 =3+. xo+x~ (1) (xo+zo)2 =4+yo+y~ (2)

(y 0 + XO)2 = 5 + Zo + z~ (3)

CQng 3 phtrong trinh cuaM nay ta co:

(xo +yo +zo)2 =12+(xo +yo +zo) (*)

Giai phuong trinh b~c 2 ta co : Xo + Yo + Zo = 4 ( 4 ) hay Xo + Yo + Zo = -3 (5)

a) Til (1), (4) ta co: x, =.!2, til (2), (4) ta co :yo =g, til (3), (4)

.. . , 9. 9

, 11

ta co Zo =-

9

b) Til (1), (5) ta co: x, = -6, til (2), (5) ta co: Yo = -5 , til (3), (5)

5 5

-4 taco Zo =- 5

K~t lu~n :

{( 9 9 9) (-5 -5 -5) }

(x,y,z) e -,-,-, '-,-,- ,(t,O,O),(O,t,O),(O,O,t),teR .

13 12 11. 6 5 4

G

TRtlt7NG ,HurEN LE QUi' eON,·rtNH KHANH HOA

k:;:)

C7 TIm gia tri Ion nha't eua bi~u thirc: P = x3y, + y3x , vdi x , y E R va x, y thoa di~u kien: .x2 + xy + y2 = 1.

Bai 2.

TIm ta't ea cac tam giac ABC co dQ dai 3 canh la cac s5 nguyen duong, khong e6 ude chung va thoa man dAng thirc :

eotg2 A + 4eotgi B + 9cotg2 C :::: (6p)2

2 2 2 7r '

Vdi p, r l~n IU<;1t la mi'a ehu vi va ban kinhdudng tronnQi tie'p tam giac ABC.

Bai 3.

La'y cac canh BC, CA, AB cua tam giac ABC lam day, dung ra phia ngoai 3 tam giac vuong can MBC, NCA, PAB . ChUng minh rAng 3 du()ng thing AM, BN, CP d&ng quy va hai tam giac ABC, MNP co cung trongtam,

Bai 1.

GQi a la mot gia tri cila bi~u thttc P, nghia la h~ sau e6 nghiem :

{x2 +Xy+y2.::::l (1) x3y+y3x::::a (2)

f)~t u = x2 + y2; V = xy (di~u kien : u ~ 2v). H~ tren nrong duong vdi

{u +. v = I, uv-<:::a

suy ra u, v Ia nghiem eua phirong trlnh : X2 - X + a = O. Nghiern thea man . 2

u ~ 2v<=> a ~ -.

9

Yay: MaxP = .2 khi x = y = ± ~.

. 9 v3'

BBi 2.

P ABC

Ta e6: - = cotg-cotg-eotg- va

r 222

62

63

ABC 'A B C

cotg-cotg-cotg- = cotg- + cotg- + cotg- ,

2 2 2 2 2 2

(*)

Tir giii thie't, ta c6 :

2A 2B 2C A, B C 2

49(cotg -+4cotg -+9cotg-)=36(cotg-+cotg-+cotg-) (1)

2 2 2 ' 2 '2 2

Ap dung BDT Bun-nhi-a-cop-ski:

36( cotg A + cotg B + cotg C )2 S 49( cotg2 A +4cotg2 B + 9cotg2 C) (2)

2 2 2 ' 2 2 '2

ABC

cotg- 2cotg- 3cotg-

Da'u bhng xay ra ~, 2 = 2 = 2

6 3 2

\\

,

Ke't hop vdi (*), ta dtt<;Jc :,

A cotg-=7 2

B 7 cotg-=- 2 4

C 7 cotg-=-

2 9

'A 7

SIn =-.

25

'B 56'

~ SIn =-

65

'C 63

SIn =-

65

Ad d' h li h' ' 25a65b65c

Pungm lamsm:--=-=~

" 7 56 63

Ta chon : a =13, b = 40; c = 45 ,

Ke't lu~n : !l ABC c6 3 canh

(a; b; c) = (l3k; 40k; 45k) vdik E N*,

~ {40a=13b , 45a=13c

Bai 3.

a) Dung ra phia ngoai A ABC, 3 hlnh vuong BCC,B" ACC2A2, ABB2A, thl M, N,pMn hrot la tam 3 hlnh vuong n6i tren,

90"

x« phep quay Q8 : A ~ B2

B, ~ C ~ AB, = B2C va AB, 1. B2C (1)

GQi D la trung di€m ella AC, ta e6 DM, DN I~n ht<;1t la d1tong trung 'blnb ella !lACB,. !lACB2 ' Ke't hop vdi (I). ta e6 :

DP = DM = .!_ AB, = .!_ B2C va DP 1. DM

2 2

X.( he Q90" ',P ~ M

er P ep quay f) -"7

N ~ A ~ PN = MA va PN 1. MA

~ MA ia ~1tong eao ella!l MNP,

Hoan toan nrong tu, ta chrrng minb dlt<;1e BN, CP la cacdinrng eao ella !lMNP,

Do d6AM, BN, CP ddng quy tai tnrc tam cila ~ MNP. GQi Q, R l~n 111<;1t la trung diim PN, AM, va G = MQ (l BD.

Theo chtrng minh tren, Q~oo : PN ~ MA, nen

Q~OO : Q~ R => DQ = DR va DQ .1 DR. (2)

Vi DR la du'CJng trung blnh cua A AMC => DR II MC va DR = _!_ MC 2

Ma: MC = MB va MC .1 MB nen DR .1 MB va DR = _!_ MB. (3)

2

Til' (2) va (3), ta e6 DQ II MB va DQ = _!_ MB.

. 2

Theo dinh li Ta-let : QD = QG = DG = _!_ va G E BD, G E MQ va BD,

. BM GM BG 2

MQ la trung tuyen cua AABC, AMNP, suy ra G la trong tam AABC va AMNP (dpcm),

64

OLYMPIC MON ToAN - 5

65

TRU'CrNG THPT CHUYEN IRAN HlING BAO. TiNH BiNH THUAN

,LJ

C7 GiAi h~ phirong trinh

{ 6X2 = y(l +9X2.) 6y2 = z(l +9y2)

6z2 = x(l +9z2)

(1) (2) (3)

. ~~i1J. '

U' Cho a,b,e » Othoa man a+b+e=l.

T' . , . nho hI. , T ab be ea

im gra tri n at eua = +. + '.

. e(b + e) a( e + a) b( a + b)

Baj 3.

, Cho tam giac ABC co cac canh a, b, e va cac goc thoa man B =2A, C =4A.

'T'hS R2(1 1 1) ri RIa ba kinh d , .. ;.

in = -y + 2 + 2'" .voi a an Ill, irong tron ngoai tiep

abe

tam giac ABC. Baj 4!

Cho tam giac ABC, dung ra phia ngoai tam giac do ba tam giac can ~ACIB,~BAIC,~CB)A co cac canh day AB, BC,CA va goc a day 1ft a.

Chung minh ba dirong thfutg AA),BB),CC) d6ng quy.

Baj 1.

Hu'ONG DAN GIAI

Taco

6x2 =y(I+9x2)~y=3x2(2-3y) Y = ~' : khong la nghiern cua h~ phirong trinh.

y:;t: 2 : til (*) ta co x2 = y'

3 3(2-3y)

Ta co _a_ = _b_ = _c_ = 2R sin A sin Bsin C .

R 1 R 1 Rl

=> a- = 2 sin A 'b = 2 sin B ' -z = 2 sin C

Phuong trinh nay co nghiem ~ y ~ 0 ~ 0 ~ y < ~ .

. 3(2-3y) 3

2 2

Tuong tu : 0 ~ x < -, 0 ~ z < - .

3 3

Ta co : x = y = z = 0 1ft nghiem M phirong trinh

V6i x.y.z o O , tir(1): 1..= 6x 2 ~l~y~x x 1+9x

tir (2) : z ~ y

tir (3) : x ~ z

1 => y~ x s z s y =>x =y=z=-. 3

V~y nghiem cua M phirong trinh 1ft (0; 0; 0), (~; ~; ~) .

Hai 2.

Ta co ( [iib + [bC + rca)2 v7 V-a Vb

=( ~~b+C+~. bC.Jc+a+ ~.Ja+bf

V~· va(c+a) V~

~ ( ~b + bc + ca )(b+c+c+a+a+ b) = 2T c(b + c) a( c + a) b( a + b)

nen:

(~+~+ ft)2 ~3(a+b+c) =3. => 2T ;?:: 3 => GTNN cua T la l.

, .. 2

, d d khi [ab [bC rca . b 1

GTNNnay at UQ'c 1 V7=V-a=Vb~a= =c=3'

Hai3.

66

67 "

1 1 1 1

~S= 4 (sin2 A + sin2B + sin2C)

1

= 4 (3+cotg2A+cotg2B+cotg2C)

Trong MBC co cot gAo cot gB +cot gBC.tgC + cot gc. cot gA = 1 cotg2a-l A

va ta c6: cotg2a = . nensuy ra:

2eotga.

cot g'o = 1+2cotga.cotg2a

~ S = _!_ [3 + 3 + 2{ cot gAo cot g2A + cot gB. cot g2B + cot gC. cot g2C) ] 4

1

= 4[ 6 + 2{ cot gA.cot gB + cot gB.cot gC + cot gC.cot gA) ]

= !(6+2)=2.

Hai 4.

Khong mAt tinh t5ng quat, gia sir A ~ B ~ C .

Truong hQ'P 1: A + a < 1800• G9i M. N, P Ian hrot Ia giao di~m cua AA" BBI, CCI voi BC, CA, AB. Ke BH, .L AAI, CH2 .L AAI . Ta c6:

MB_ -MB_ BH, MC - MC -- CH2

SABA, c.BA,.sin(B+a)

=- SACA, =- b.CA,.sin(C+a)

csin(B+a)

=

bsin(C+a)

Tirong tu,

NC asin(C+a) PA bsin(A+a)

==- ,==-

NA csin(A+a) PBasin(B+a)

---

MB NC PA .).

~=.=.-==-I~AA"BB"CC, dongquy.

MC NA PB

-- 0 . ).

Truong hQ'P 2: A + a = 180 . AA" BBI, CCI dong quy tai A.

Truong hop 3: A+a > 1800• •

csin(B+a)

-=-=---=-

bsin(C+a)

MB MB.

MC Me

NC _ NC _ asin(C+a)

=---

NA NA -csin(A+a)

PA PA bsin(A+a)

-=-=-=-

PB; PB asin(B+a)

---

MB NC PA

:::>=.___:.==-}

MC NA PB

~ AA),BBI'CC) dAng quy.

TRUCmG THPT CHuvEN LUONG THEvINH. TiNH DONG NAI

Bai 10

Cho tam giac ABC co dQ dai 3 canh nrong irng la a, b, c. 0 la tam duong tron ngoai ti~p ; H ta tI'\IC tam. R ta ban kinh duong tron ngoai ti~p. Gia su OH c~t CB va CA tai P va Q.

Chung minh r~ng di~u ki~n dn va du d~t(r giac ABPQ nQi ti~p la: a2 +b 2=6R2.

Bai2.

Cho p ta s6 nguyen t6. Chung minh rkg s6:

11 .... 122 .... 2 ........ 99 ... 9 -123456789

~'---v--' ~

p p p

chia h~t cho p. Bai 3.

GQi s, t, u, v ta cac s6 n~m trong (0;;) sao cho:

s + t + U + v = 1t.

Chimg minh r~ng:

.J2 sin s - 1 .J2 sin t -1 .J2 sin u -1 .J2 sin v - 1 0

----+ + + ~ .

coss cos t cos u cos v

68

69

HllONG DAN GIAI

Bili 1.

Ta co cac nh~n xet sau day: .

1) Trong tam giac ABC, ta co : CH = 2R.cosC 0cH = IB - AI

2) CO.l OH ¢:> ABPQ nQi tiSp. Chung minh :

1) GQi K la trung di~m AB ; ta co:

--

CH = 20K = 2R. cosBOK = 2R.cosC.

Ta co 0cH =liTcA-OcAI =1(900 -A)-(900 -B)I =IB-AI.

2) Ke tiSp tuyen voi duong tron (ABC) tai C; cit AB tai T ; Khi do ta co fCB = CAB, OC.l CT nen ttl giac ABPQ nQi tiSp, do do

---

CPQ = CAB = TCB ¢:> CT II PQ ¢:> CO .1 OH .

Ap dung vao bai ta co :

Ttl giac ABPQ nQi tiSp

¢:> CO.l OH ¢:> CO = CH.cos(HCO) ¢:> R = 2RcosC.cos(B-A) ¢:> cos2A+cos2B= -1 ¢:> Zsin ' A + 2sin2 B= 3 ¢:> a2 + b2 == 6R 2

Bai 2.

Xet p = 3 : d~ dang chimg minh. Xet p * 3 : Ta co:

p-I p-I p-I

n = ~)08P+k +2~)07P+k + +9~)Ok-C

k=O k=O k=O

= ~(1 OP -1)( 108p + 2.1 07p + ..... + 8.1 OP + 9) - C =~(109P +108p + .... +10P -9)-c

p la iroc cua n khi va chi khi 9p Ia troc cua 9n (vi ta xet p khac 3).

Ta se chirng minh r~ng:

109p + 108p + .... + lOP - 9 - 9c chi a hSt cho 9p.

Vi 9 + 9c = 1111111110 = i 09 + 108 + ..... + 10 nen ta se chirng minh (109P + 108p + .... + lOP) -( 109 + 108 + ..... + 10) chia hSt cho 9p.

l.

Theo dinh If Fermat nho, ta co: rOmp =(10mr = 10m (modp) lOmp = 1 om (mod 9)

Mil (9;p) = 1 nen IO'" == 10m (mod 9p). Suy ra dpcm.

Bai 3.

D~t a = tgs; b = tgt; c. = tg u ; d = tgv. Khi do a, b, c, d > o. Ta

co:

. a+b c+d s+t+u+v.=7t=>tg(s+t)+tg(u+v)=O=>-o _+ __ o =0 l-ab I=-cd

<=> a + b + c + d = abc + abd + acd + bed Suy ra:

( a + b) ( a + c) ( a + d) = a 2 (a + b + c + d) + abc + abd + acd + bed

=(a2 +1)(a+b+c+d)

a2 +1 (a+c)(a+d)

=> _0_ = ~__:_~--L.

a+b (a+b+c+d)'

Ap dung bAt ding thirc Bu-nhi-a-kop-ski: 2(a+h+c+d)2 =

[a2+1 b2+1 c2+1 d2+1] =[( a + b) + (b+ c) + (c + d) + [d +a)] _0- +- .. - + -- +a+b b+c c+d d+ a

~ (.Ja2 +1 +.Jb2 +1 +.Jc2 +1+.Jd2 +1)

Suy ra:

.Ja2 +1 +.Jb2 +1 +.Jc2 +1 +.Jd2 +1 ;S;.J2(a+b+c+d) Tirc Iil:

_1_ + _1_ + _1_ + _1_ ;S; .J2 (tgs + tgt + tgu + tgv) cos s cos t cos u cos v

Day Iil dieu phai chimg minh.

70

. mUONG THPT CHUYEN LE QUY eON. TiNH BA RIA - VUNG TAu /Bro/}

(~

. Ghii phirong trinh : ~ 6x + 1= 8x3 - 4x -1 .

Bai 2.

Ch ,~ ,.. he - x Y Z 9

o cac so nguyen x, y, z t oa man: ---+- = .

Y z x

Chung minh r~ng : ~ E Z .

Bai3.

Cho ~ABC c6 A < C < 1t va cac s6 thuc m, n, p thoa man:

2 .

~+ __ n_+_p_·=O.

cosA . B cosC

SIn-

. 2

Chung minh phuong trin~:. .

mx +nx+p=O

c6 nghiem x E (0;1). Bai 4.

Cho MBC thoa man : cos A. cos B. cos C = _l. 8

a) -Chirng minh t3n tai met g6c cua ~ABC c6 s6 do nho nhdt Ifl 1200•

b) GQi M, N, P Ifl di~m d6i xirng' cua A, B, C qua BC, CA, AB.

Chung minh M, N, P thang hang.

HUONG DAN GIAI

Bai 1.

each 1: ~6x+l =8x3-4x-l<=>6x+l+~6x+l =(2xl+2x (1)

Phuong trinh c6 dang : f(~6x + 1) = f(2x) voi f (t) = e + t la ham s6 d3ng bien trong 1R . V~y phuong trinh (1) nrong duong voi:

~6x + 1 = 2x <=> 8x3 - 6x = 1.

Nhan xet :

Neu Ixl > 1 => 4x2 -3> 1 => 18x3 -6xl = 21xl(4x2 -3) > 2.

71 Jo

lfCLN(x,y,z)=d=>x=dxo;y=dYo;z=dzo, voi xo,yo,zo EZ va UCLN(xo'yo'zo) = I. Khi do:

~-~+~=9 (1) ; ~=d1xoYozo Ez<;::;>1xoYozo EZ. Yo Zo Xo

Nhan xet : bQ (xo,yo,zo): xoYozo = ±I khong thS thoa man (1) nen xoYozo -::t:. ±I. G9i p lit mot uoc nguyen t6 bAt ki cua XoYoZo .Vi

(I) ¢:> x~zo - y~xo + z~y 0 = 9xoYozo (2)

va (x.; Yo' zo) = 1 nen p chi lit uoc cua dung hai trong 3 s6 xo' Yo' Zo .

Khong giam t5ng quat, gia sir p lit iroc cua xo; Yo vit khong lit uoc cua zoo G9i m, n IAn hrot lit s6 mu cua p trong phan tich tieu chuan cua x, va Yo (m,nEZ+).

Ta xet hai truang hap:

a)N~u n z Zm+J : ta co p'""" la uoc cua z~yo,y~xo,9xoyozo nentil (2) suy ra p2m+1 lit iroc cua x~zo' mit x~ khong ehia h~t cho p2m +1 nen nen p lit mrc cua z, (mau thudn).

b) N~u n ~ 2m-1 ¢:> n+ I ~ 2m : ta co pn+llit uoc cua

2 2 9

xozo,Yoxo' xoYozo

nen til (2) suy rap'" +1 lit iroc cua z~Yo; mit Yo khong chia h~t eho pn+1 nen p lit uoc cua z, (mau thudn). V~y n = 2m nen 3m hi. s6 mil cua p trong phan tieh tieu chuan cua xoY oZo .

nen nghiem cua (I) n~u co phai thuoc [ -I; 1 ]. f)~t x = cos t, t E [0; 1t]. ( I) tro thanh:

3 1 1 .. 1t o21t

4cos t -3eos t = - <;::;> cos3t = -¢:> t = ±-+ k~,(kE Z)

2 2 9 3

S (I)' A h'A us { 1t 51t· 71t}

uy ra co t~p ng iem a: =. eos9';eos9;eos9 .

, ~ =2y=>{8y3 ::;:6x+l.

each 2 : f)~t 8x3= 4x + 2y + 1 .

Tnr 2 phuong trinh ta duoc : 8(y3 - x") = 2(x - y) ¢:> x = y. V~y(l) ¢:>8x3-6x=1.

Cac biroc ti~p thea giai nhir each I.

Bii 2.

72

73 ).

k

Suy ra xoYozo = Dptmi (voi Pi, i = l.k . la cac iroc nguyen t6 cua ;=1

xoyozo)' V~y ~xoYozo E z ,

Chu 'T 'hA hi A . J.. bA b ( ). he x Y z 9 '

u y: a co t e c Ira vo so 9 a x, y, z t oa ---+- = ,VI

Y z x

du: x = t , Y = 3t, z = 9t voi t la s6 nguyen tuyy khac 0, va ~. = 3t .

Bai 3.

Tnroc h~t, voi ~ABC th6a A < C < 7t ta co: 2

1) cos A. cos C =.!_( cos(A - C) + cos(A + C») <.!_ (1-'- cos B) = sirr' B 222

7t B 7t 7t. . B

2)0 < A < C < - ~ 2A + B < 7t ~ 0 < - < - - A < -~ sin - < cos A

2 2 2 2 2

f)~t u = cosA;v = sin ~; w = cosC ~ 0 <: < 1,0 < uw < v2 (1)

va f(x) = mx ' +nx+p.

NJ.. 0" h' J.. b" r • hanh rn n 0

eu P = : gla t let al toan tro t C1 - + - = .

u V

Neu rn = 0 thi n = 0; f(x) la "da thirc khong" nen co nghiem x E (0;1).

N~u rn:;cO~-E._= V ~f(x)=x(rnx+n) c6nghi~rnx=-E._=~E(0;1).

rn u rn u

Neu p:;c O. Ta bien d6i gia thiet :

rn n P u (V2 V J uw - v2

-+-+-=O~- rn-. +n-+p -. p=o

u v W v2 . u2 U v2w

2

~ f'( v ) = uw - v f(O).

u uw

V~y: f(V).f(O)= uw-v2 [f(O)f <0. (do(l)vaf(O)= p s O)

u uw

Suy ra f(x) co nghiem x E ( 0;:) < (0; 1) nen f(x) co nghiem x E (0;1).

Suy ra:

MN = AN _ AM = 2c ( co~ A _ CO: B ) ~ -:- (I ~ 2b c:s C ) U

MP = AP-AM = 2b( co: A _ cO;C)U_(I+ 2CC:SB)~ V~y: M, N, P thiing hang ¢:> MN,MP cung phirong. Di~u nay nrong dirong voi:

Rai 4.

a) Vai tro A, B, C nhtr nhau va til gia thi~t suy ra MBC til nen khong giam tBng quat, gift sir A> 90' > B ~ C .Taco :

3 3

cosAcosB.cosC = -- => - = -cosA(cos(B-C)-cosA)

8 ,4 '

, 3 , 3 ' 1

=> - s; -cosA(l-cosA) => cos! A-cosA-- ~ 0 => cosA s;--

4 4 2

V~y A ~ 120'. Diing thirc xay ra khi : B = C = 30' (thoa man: cosI20°.cos30°.cos30° = -~ ).

8

b) GQi H, K, L Ian hrot la hinh chieu cua A, B, C tren canh d6i di~n

(ta v~n gift sir A til). D~t AB = U, AC = ~ , ta co:

AH - AB BH - - c.cosB BC _ - . c.cosB (- -)

- + -u+ ,-u+ .v-u

a a

a-c.cosB - c.cosB-

= u+ v

.. a a

b.cosC - c.cosB-

= u+ v tdo a e b.cos Ca-c.cos B).

a a

BK BA AK - c.cosA - CL CA AL - b.cosA-

= + = -u+ v, = + = -v+ u.

b c

M, N, P la di~m d6i ximg cua A, B, C qua BC, CA, AB nen ta co: - 2bcosC - 2ccosB - -.

AM= u+ v;AN

a a

AB 2BK .- 2c.cosA-

= + =-u+ v·

b '

AP AC 2CL - 2b.cosA-

= + =-v+ u,

c

74

75

2C( CO~A _ CO:B}2b( c":A _ CO:C)~(l+ 2bC:SC)(1+ 2CC:SB)

~ 4(acosA - bcos B)(acos A -c.cosC) = (a + 2bcosC)(a + 2c cosB) ~ 4[a cosA -(bcosB + c.cosC)]cosA = a + 2(bcosC +c.cos B)

9 4[acos A - (bcosB + c.cosC)]cos A = 3a

sinB sinC

~ 4[cosA-(-.-cosB+-.-cosC)]'CosA = 3

smA sin A

~ 4[-cos(B+C)-cos(B-C)]cosA = 3 3 .'

~ cosA.cosB.cosC = --.

8

D~ng thirc cu5i dircc cho a gia thiet nen ta c6 di€u cAn clurng minh.

iJilNG THPT CHuvEN L~HONG PHON!>, TP. HO CHi MINH· (Bili

Tim tAt ca cac ham s6 f: R+ ~ R+ ( R+ chi t~p hop cac s6 thirc dirong) thoa man d6ng thai 2 di€u ki~n :

(l) V' x, Y E R+, n~u x ~y thi f(x) ~ f(y)

(2) f(x.y). f( f(y)) = 2006, V' x, y E R+.

X

Bai 2.

Cho m, n la hai s6 nguyen dirong phan biet eo (m, n) = d. Tinh (2006m + 1, 2006n + 1).

~r, (Ki hi~u (m, n) chi iroc chung Ian nhdt cua m va n).

~ .

Tim gia tri Ian nhdt cua bieu thirc

x3 + y3 + Z3

F =. voi x, y, z thuoc doan [1003; 2006].

xyz

r".

10'·4'

, al

Cho tam giac nhon ABC nQi ti8p trong duong tron tam O. Duong tang AO e~t canh BC tai D. Tren hai canh AB va Ac IAn hrot lAy di~m M va di~m N sao eho DB = DM va DC = DN: CM va BN cit nhau tai E. G9i H va K la true tam cac tam giac EBM va ECN. Chung minh HK vuong g6e voi AE.

Hu'O'NG DAN GIAI

Dai 1.

(1)

Gia sir ham s6 f: R+ ~ R+ d6ng thai thoa man hai di~u kien: (1) 'V X, ye R+, n~u x ~ y thi fix) ~.f(y)

(2) f(x.y). f( fey) ) = 2006, 'V x.y e R+.

x

Til (2) Ifty x = 1 ta co:

fi(fi( » = 2006 'V e R ". (3)

Y f'(y) , y

'V x, y e R+, neu x> y thi til (1) ta co f(x) ~ fey) => f(f(x»~· f(f(y» 2006 2006

=> -- ~ -- => fey) ~ f(x) => f(x) = fey)

f(x) fey)

=> :3 a e R+ sao cho f(x) = a, 'V x e R+ . (4)

Til (3) va (4) => a = 2006 ~ a = .J2006, 'V xe R+ a

=> f(x) = .J2006, 'V x e R+.

NguQ'C lai n~u f: R+ ~ R+ va f(x) = .J2006 , 'V x e R+ thi ra rang (1) va (2) d~u diroc thoa man.

Dai 2.

D~t r = m , s = n va 2006d = b. Ta co:

d d

(r, s) = 1, 2006m = b'. 2006" = b', b chgn.

Ta tim g = (2006m + 1,2006" + 1) = (b' + 1, b" + 1). Ta co:

{bl" + 1 : g =>{br = -1 (modg) b' + 1 : g b' = -1 (mod g)

b' + 1 : g => b' + 1 = kg (k e Z) => kg - b' = 1 => (b, g) = 1

(r, s) = 1 => :3 u, V e Z sao cho ru - sv = 1. (r, s) = 1 nen r, s cling le hoackhac tinh chgn le.

Truong hop r, s d~u le:

Khi do b' +- 1 va bS + 1 co b + 1 la mQt uoc chung, suy ra g:(b+I)

{bnl =(_I)U (modg)

Til (1) => . •

bSV = (-If (modg)

Ma r, s d~u le va ru - sv = 1 nen u, v khac tinh chgn le. Do do:

76

(2) (3)

(4)

).

b'" + bSV == (_I)U + (-lr == 0 (mod g)

(2)

::.:::> bSV + I + bSV = bSV (b + l) : g ::.:::> b + 1 : g.

Tir (4) va (5) ::.:::> g = b + 1. Truong hQ'P r, s khac tinh ch~n le:

{brS = (-IY (modg) .

(1) ::.:::> - . . ::.:::> 2.brs == (_I)r + (-It == 0 (mod g)

b" == (-IY (modg)

• (2)

::.:::> 2.brs : g ::.:::> 2 : g.

Ma b ch~n ::.:::> b' + 1, b" + 1 Ia cac s6 le ::.:::> g le.

Tu (6) va (7) ta co g = 1. V~y:

(2006m + 1, 2006" + 1)·~ 2006d +1 n~u m, n cling le ;

. . d d

(2006m + 1, 2006" + 1) = 2006d + 1 n~u m, n khac tinh ch~ le, d d

Bili 3.

Vi x, y, z co vai tro nhu nhau nen ta co th~ gia sir 1 003 s x ~ y ~ z ~ 2006.

D~t y = kx va z = hx (1 ~ k ~ h ~ 2) ta diroc:

x ' + k3x3 +h3x3 1 + k3 + h3

A= =---

x.kx.hx kh

Ta clnrng minh

hk 2~

(2) <=> 2 + 2k3 + 2h3 s h + hk3 + h.23 <=> (2 - h) + k3(2- h) + 2h(h2 - 22) ~ 0

<=> (2 - h)[l - 4h + k3 - 2h2] $; O.

Vi 2 - h ~ 0, 1 - 4h s 0 va k3 - 2h2 s 2k2 - 2h2 s 0 nen (*) dung. M~t khac ta co:

I+k3 +23 k3 -IOk+9 (k-'-I)(k2 +k-9)

--- -5= = ~O.

2k 2k 2k

Tir (1) (2) va (3) ta co : A ~ 5. Lai ~o khi x = y = 1003, z = 2006 thi A=5.

V~y gia tri Ian nhdt cua A bang 5.

(5)

(6) (7)

(1)

(2)

(*)

(3)

77

Bai 4.

G9i P vA T IAn hrot IA trung

, di~rn cua BM vA AB

=> DP 1- AB (do OM = DB) vAOT 1-AB.

=> OT IIOP

=> ~: =: =2(1- :)

AP AD'

= 2--. -=2--.

AT AO

Tuong t1,1' ta cling c6:

NC=2_AD.

AC AO

Ood6 MB = NC AB AC => MN II BC => AE di qua trung diem J cua MN.

G9i (Yt) IA dirong trim tam I, dirong kinh AE va (Y2) IA duong trim tam J, duong kinh MN.

G9i F, G IAn hrot la hinh chieu vuong gee cua E tren AB vA AC. G9i U IA hinh chieu vuong g6c cua M tren BN va V lA hinh chieucua N tren CM. Tac6:

PH/(y,) = HE.HF va PH/(Y2) = HM.HU

. ma M, F, U, E cung thuec dirong trim dirong kinh EM nen

. HE.HF = HM.HU => PH/(y,)= P~/(Y2)

Tirong t1,1' ta c6: PK/(y,) = KG.KE = KN.KV = PK/(Y2)' Suy ra KH IA true ding phirong cua 2 duong trim' (y,.), (Y2)' Suy ra HK 1. IJ => HK 1- AE.

TRU'CmG IHPJ CHUYEN IRAN f)AINGHiA. IP. HO CHi MINH

Bai 1.

Giai phuong trinh:

(x - X2)(X2 + 3x + 2007) - 2005x.J4-4x = 30{/x2 + x -1 + 2006.

78

·79

Bai 2.

a) Cho x, y > 0, x + y ~ 1. Tim gia tri nho nhAt cua.bieu thirc:

. 9 48

P = 51x+23y+-+-.

. x 7y

b) Cho a, b, c > O. Chung minh bAt dftng thirc:

(a2 +ab + b2)(b2 + be+ C2)(C2 +ca + a") ~ (ab+ be -ica)",

Bai3.

Cho a, b, c lit dQ dai ba canh cua mot tam giac va R lit ban kinh duong tron ngoai ti€p cua tam giac d6. Chung minh r~ng:

a3 + b3 + c3 + abc

_______ ~ 4R 2.

a+b+c

Bai 4.

Cho t~p hQ'P A = {al,a2,aJ ... ,a2006} .Ta xdy dung cac t~p hQ'P B, C, D

nhu sau:

B={bpb2,b3, ... ,b2006} voi bi= ai+2ai+1 (i = I, 2, ... , 2006),a2007 = a,

C { } , . bi + bi+1 (. 1 2 2006) b b

= CI,C2,C3,· .. ,C2006 val ci = 2 1 = , , ... , , 2007 = I

D {d d d d } ,. d c, + Ci+1 (i 1 2 2006)

= i' 2' 3' ... , 2006 val i::: 2 1 = , , ... , , C2007 = c,

BiSt A = D va a, = I. Tim a2,a3,· .. ,a2006·

HUONG DAN GIAI

Bai 1.

Phuong trinh da.cho nrong dirong voi:

~--

(x ' + X _1)2 + 2005(x+ ~)2 +301x2 + x-I = 0

{X2 + x -1 = 0 . -1-.[5

<=> ~x=.

x + :../1 - x = O' 2

Bai 2.

a) Ap dung bAt dftng thirc Co-si:

9 . 48 .

P = 2(x +y)+(49x +-)+(21y+--) ~ 2+42+ 24 = 68.

x 7y

S .,. he h~ , p}' 68 khi 3. 4

uy ra gra tn non at cua a ,IX = -;:; ; y = -;:; .

b) Ap dung bfit d~ng thirc Co-si - Svac:

(ab+ bc +cauab+ b2 +a2) ~ (ab+M +M)2 (1)

(b' + bc+c2)(a2 +c2 +ca) ~ (ab+M +~)2 (2)

(ab+M+M)(ab+M +~)~(ab+bc+ca)2 (3)

Til (1), (2), (3) ta suy ra b&t d~g thirc can phai chirng minh.

Bili 4.

Ta co (a + b)OC + (b+c)OA +(c+a)OB t ~ 0

~ [(a + b/ +(b+C)2 + (c+a)2]R 2 + 2(a + b)(b+ c)OC.OA

-- --

+ 2(b+ c)(c + a)OA.OB + 2(c +a)(a + b)OB.OC ~ 0

~ [(a+b)2 +(b+C)2 +(c+a)2]R2 +(a+b)(b+c)(2R2 _ b") +(b +c)(c +a)(2R 2 _c2) + (c+a)(a + b)(2R 2 - a2) ~ 0

~ (a + b+c)24R 2 ~ a2(a + b)(a +c)+ b2(b+c)(b +a) +c2(c + ",)(c + b) ~ (a + b+c)24R 2 ~ (a + b+c)(a3 + b3 +c3 + abc)

Bai 4.

Taco:

a;+a; a;+a~ a;006+a~ >(a,+a2)2 (a2+a3)2 (a2006+a,)2

2 + 2 + ... + 2 - 2 + 1. + ... + 2

--.. 2 + 2 + +i 2 > b2 + b2 + + b2

- a, a2 ... a2006 -, 2'" 2006

Chirng minh tuong tu ta co:

b; + b; + ... + b;006 ~ c~ + c; + ... + C;006 ~ d~ + d; + ... + d;006 .

S 2 2 2 d2 d2 d2

uy ra a, +a2 + ... +a2006 ~ I + 2 + ... + 2006'

Do A = D (gia thiet) nend~ng,thuc xay ra. Til do suy ra:

a, = a2 = ... = a2OO6 = 1 .

80

OLYMPIC MON rOAN - 6

81

TRu'ONG THPT MAC BiNH CHI, TP. HOCH! MINH

Bili 1.

Tim nghiem dirong cua h~ phuong trinh: 2007

XI +x2 -x) =--

XI x2

2007 x2 +x) -X4 ---

x2 x)

2007

X2OO6 + X2007 - XI = ---X2OO6 X2007

Bili 2.

Tim gia tri cua m, a, b sao cho phuong trinh X 5 - m X - 33 .. 0 c6 hai nghiem XI, X2, voi XI, x21a hai nghiem cua phirong trinh

x2 +ax+b=O,

trong d6 a va b la cac s6 nguyen.

Bai 3.

Cho tam giac ABC. GQi rna; mb; me IAn hrot Ia dO dai .ba duong trung tuyen va ha; hb; he IAn hrot la do dai ba dirong cao til ba dinh A, B, C cua tam giac ABC. Chimg minh r~ng:

rna + mb + me < 1 + 1. C'

h, hb . he - 4 sin A sin B sm-

222

Bai 4.

Cho tam giac ABC va 0 Ia mot di~m nam trong tam giac ABC. Qua o ke ba dirong thing phan biet d, ; d2 ; d3, chung I~n hrotcat hai canh AB, BC tai M, N ; c~t hai canh BC, CA tai P, Q va c~t hai canh CA, AB tai R, T. GQi SI ; S2 ; S3 ; S IAn hrot la di~n tich ca~ tam giac OPN ; ORQ ; OMT va

ABC Ch' . h 1 1· 1 > 18

. ungmm: -+-+-_-.

. SI S2 S3 S

(1)

Hu'lmGDANGW

Bai 1.

f)~t Y k = ~, k = 1 ; 2007 . Vi Xk:> 0 nen Y k > O. Khi do h~

32007 .

phuong trinh dff cho dtroc viSt lai nhu sau: 1

YI +Y2 -Y3 =--

YI'Y2

1

Y2 +Y3 -Y4 =--

Y2'Y3

Y2007 +YI -Y2 =--Y 2007'YI

Ta thAy YI = Y2 = ... = Y2007 = 1 la nghiem cua h~ phirong' trinh (*). Ta chirng

minh h~ phuong trinh (*) co nghiem duy nhAt la Y I = Y 2 = ... = Y 2007 = 1 .

1 .

Ta co Yk + Yk+1 - Yk+2 = (1),Vk E {I; 2; ... ; 2007}.

Yk'Yk+1

(Quy iroc Yk+2007 = Yk)' Cong tAt ca cac phirong trinh cua h~ phuong trinh

(*) vS thee vS ta duoc:

1

(*)

Y2006 +Y2007 -YI =:---Y 2006' Y 2007

1

Tit do ta co nh~n xet sau:

Nhan xet 1: NSu Yk" > 1 thi Yk,,+1 < 1 (voi k, E {I; 2; ... ; 2007} )

That v~y: vi nSu Y k" ~ 1 thi ta suy ra:

1

Y = Y +Yk - > 1

k,,+2 k" ,,+1

Yk" 'Yk,,+1

Tit Y k ~ 1 va Y k. +2 > 1 => Y k +3 = Y k + Y k +2

II 0 n u+1 ·0

1 ---->1.

CU tiSp tuc nhir V~Y, suy ra:

Yk > 1, vk E {I; 2; ... ;ko; ko +2; ... ; 2007} va Yk,,+1 ~ 1.

Tit (1) ta suy ra VT > 2007 va VP < 2007 (mau thudn).

V6'i l~p luan nrong tu nhir nhan xet 1 ta co nh~n xet 2 nhu sau:

82

83

Nh~ xet 2.: N~u Yk" < 1 thi Yk,,+l >1 (voi k, E {I; 2; ... ; 2007} ). Tro lai bai toan ta e6:

Gia sir 3ko E {I; 2; ... ; 2007} d~ eho Yk" > 1 Theo nh~ xet 1 va nh~n xet 2 ta suy ra:

Ykn+1 < 1 ; Yk,,+2 > 1 ; Yk,,+3 < 1 ; Yk.,+4> 1 ; ... ; Yk,,+2007 < 1 . (mau thufut vi Y k +2007 = Y k ).

U II

HOM toan nrong tu ta suy ra diroe ring khong t6n tai ko E {I; 2; ... ; 2007} d~ eho . Yk" < 1

V~y YI = Y2 = ... = Y2007 = 1 la nghiem duy nhAt cua h~ phuong trinh (.). Tir d6 suyra XI = x2 = ... = X2007 = ~2007 la nghiem duy nhAt cua h~ phirong trinh dll eho.

Bai 2.

Vi phirong trinh x5 - mx - 3 = 0 e6 2 nghiem XI' x2 la nghiem cua phuong trinh X + ax + b = 0 nen da thirc x5 - mx - 3 = 0 ehia h~t eho x+ax+b e O:

Thirc hien phep ehia da thirc x5 - mx - 3 = 0 cho X + ax + b = 0 ta duoc : x5 -mx -3 = (x + ax + b)[ x3 -ax2 +(a2 - bjx+ 2ab-aj] +

+(a4 - 3a2b + b2 - m)x + a3b - 2ab2 - 3 => {a4 -3a2b+ b2 -m = 0 .(1)

a3b-2ab-3=O . (2)

Tir (2) => ab(a2 -·2b) = 3. (3)

Vi a, be Z => a2 - 2b E Z. Tir (3) => a la iroc s6 cua 3. Co 4 tnrong hop:

+ Truong hop 1: a = 1.

(3) <=) b(l- 2b) = 3 <=) 2b2 - b + 3 = 0 phirong trinh vo nghiem (loai). + Tnrong hop 2: a = -1.

[b=-l

(3) <=) -b(1- 2b) = 0 <=) 2b2 - b - 3 = 0 <=) b = ~ ,

. 2

a day, ta loai b = -1. Tir (1) => m = 5 . + Truong hop 3: a.= 3 .

Tirong nr :

rnb-R~ON rnc-R~OP.

(3) ¢:> 3b(9- 2b) = 3 ¢:> 2b2 -9b4-1 = 0 ¢:> b = 9±[D (loai). + Truong hop 4: a = -3.

(3) ¢:> -3b(9 - 2b) = 3 ¢:> 2b2 -9b-l = 0 ¢:> b = 9± ~ (loai). Thir lai, ta thdy voi a= -1, b = -1, rn = 5 thi:·

x2+ax+b=x2-x-l

va x5-rnx-3=x5-5x-3=(x2-x-l)(x3+x2+2x+3), suy ra phirong trinh x + ax + b = 0 eo 2 nghiem phan bi~t

I±J5

XI~=--

.- 4

va phuong trinh x5 - mx - 3 = 0 eo hai nghiem XI; x2 la nghiem cua plnrong trinh x + ax + b = 0 (thoa man yeu cAu cua dS bai),

V~y a = -1, b = -1, m = 5 .

Bili 3.

Taeo:

m m m . 1

_8 +_b +_c <1+ (1)

h h h- A.B.C

8 b c 4 sin- sm,- sm-

222

¢:> rna + mb + mc ~ 1 + R (2) (vi r = 4R sin A sin B sin C ).

ha hb hc r 2 2 2

Nhan 2 vS cua bdt phirong trinh voi S MBC ta dirge :

11 1

(2) ¢:> - a rna + - b m, +..,- e me ~ S MBC + pR 222

1 .1·. 1 . 3

¢:>-a (m-R)+-b (m -R)+-c (m -R)~S ( )

2 a 2 b 2 c MBC

Ta co : rna - R = AM - OA ~ OM .

TiI (3) ta co:

VT ~ _!_a OM +_!_ bON +_!_c OP

2 2 2

¢:> VT s SOBC ,+ SOCA + SOAB =S ABC = VP . V~y (1) duoc chirng minh.

84

85

Bai 4.

A

D~t: S4 =S~OMQ' s, =S~OTP' S6 =S~ORN

=> S4,SS,S6 = .!OM.ON.OP.OQ.OR.OT.sin04.sinOs.sin06

8 .

= i OM.ON.OP.OQ.OR.OT.sinO,. sin O2, sin 03 = SI,S2 ,S3

M khac: ~S S S S S S· S, + S2 + S3 + S4 + Ss + S6 ~ ~t ac: '2 3 4 S 6 :S 6 :S 6

1 1 1 18

=>-+-. +-~-

S, S2 S3 S

Ddu "=" chi xay fa khi S, = S2 :::: S3 = S4 = S, = S6 = ~, hay

d.; d2; d) la 3 duong trung tuyen va 0 la trong tam cua ~ABC.

[~]

TRUONG THPT NGuylN THu'Q'NGHIEN. TP. 'HO CHi MINH

Oail.

, Cho tam ~iac yuong ABC ~6i, canh h~,y~n BC co dQ d~i hi, a. Chi a BC thrum 101 phan bang nhau. KhI do tam giac ABC dU'Q'c chia thrum 101 tam giac nho va tam giac a chinh gitra co goc tai dinh A bang a . GQi h Ia

khoang each til A d~n BC. Chtmg minh rang tga = lOJh

2550a

Oai 2.

Tim tAt ca cac sf> thuc m sao cho h~ phirong trinh sau co nghiern thirc x, y, z :

Oai 3.

TAng ctla m sf> duong cMIl khac nhau va n sf> dirong le khac nhau la 2001. Tim gia tri Ian nhdt cua bieu thirc A = 5m + 2n.

Oai4.

Chung minh r~ng n~u a, b, c la dQ dai 3 canh cua mQt tam giac co

.. ' ,13 2 2 2 1

chu VI bang 1 thi -:s; a + b + c + 4abc < - ;

27 2

HUONG DAN GIiI

Oai 1.

GQi AMN IA tam giac a chinh giua co dien tich hi S, ta co

50a a, S 1 MN·h . h.a

BM=CN=-,MN=-va =- . =-.

101 101 2 202

GQi I IA trung di~m BC. Trong tam giac AMN, ta co :

AM2 + AN2 = 2AI2 + MN2 vdi AI= ~ :::) AM2 + AN2 = 510 la 2

2 2 10201

VA sin a 2S. 2AM.AN = lOlh

ay tga= cosa = AM.AN· AM2+AN2 -MN2 2550a

Oai 2.

86

87

"

Tac6:

(*) <=> {(.JX +1 +.Jx-l)+(~y +1 +~y -1)+.("/Z+ 1 +~) = 2m

(.Jx+l-.Jx-l)+(~y+l-~y~I)+(JZ+i -~) = 2

D~t u=.Jx+l+.Jx-l; v=~y+l+~y-l; w=.Jz+l+~. Do x.y.z ~ 1 nen u, v, w ~..fi, Ngiroc lai n~u u, v, w ~..fi ta c6

.Jx + 1 - ~ =?:.. ~ .rx+i. = '!('u +?:..) ~ x = .!(u + ?:..)2 _; 1

u 2 u 4 u

~ x = : ( u2 + ~ ) ~ 1.

Tuong tu y, Z ~1. V~y bai toan da cho tro thanh : Tim tAt ca cac sf,

- {U+v+w=2a

thirc m sao cho h~ sau c6 nghiem u, v, W ~..fi: . 1 1 1 (I)

-+-+-=1

u v W

Di~u kien cAn:

Gia su (I) c6 nghiem (u, v, w). Theo BBT Bu-nhi-a-cop-ski, ta c6:

2m=(u+V+W)(.!+.!+_!_)~9~m~ 9.

u v w 2

Di~u kien du:

GiA su m ~ ~ , ta chirng minh (I) c6 nghiem. LAy w = 3( thea man

w~..fi).

{u+v = 2m-3 (I) <=> uv = 3(2~ - 3)·

Khi d6 u, v IA nghiern pt X2 - (2m -3)X + 3(2m - 3) = 0 . 2

~ = (2m -3)(2m -9)~ O~ u, v = 2m-3±~(2m-3)(2m-9)

2

Ta c6 (2m - 3)(2m - 9) = h(h + 6) < (h + 3i < (h + 6 - 2..fi)2 ,

h = 2m - 9 ~o ~ 2m .... 3-2..fi »(2m -3)(2m -9)~u, v ~..fi. Nhir v~y (I) c6 nghiem u, v, w ~..fi .:

VI v*y nr gia thi~t suy ra 2001" m' +m +0' = (m + ~)' +0' - ~

(1)2 1

=> m +"2 + n 2 ~ 2001+ 4 .

Ta e6 A = 5m + 2n =

(1)

T6m lai m ~ 2. Iii tAt ca cac gia tri thoa yeu du bai toan.

. 2 .

Rai 3.

T6ng cua m s6 dirong chin khac nhau khong be han

_ m(m +1) 2

2 + 4 + ... + 2m - 2. = m + m .

2

T6ng cua n s6 dirong le khac nhau ~ 1 + 3 + ... + (2n -1) = n2 .

5(m + ~)+20 - ~,; (5' +2'{ ( m +H +0'] -% (2)

Til(l)vli(2)suyraA~ 29(2001+ ~)-% =>A~238,407 (3)

Vi m, n nguyen duong nen A = 5m + 2n Iii nguyen dirong. Vi th~ til (3) ta e6 A s 238 .

I. {Sm+2n=238(S)·

. Dau "=" xay ra khi .

m 2 + m + n 2 = 2001 (6)

Giai (5) : (mo, no) = (0, 119) Iii mot nghiem rieng ella (5). Do d6 (5) e6 vo s6 nghiemIm, n) xac dinh boi:

{m =2t

(t E Z) n = 119-5t

Vi m, n > ° nen ta e6 {2t > 0 <=>.{t > ~19 <=> 0 < t < 23, t E Z .

119 - 5t > 0 t < -

5

Thay vao (6) ta duoc :

c 4e + 2t +(119 -5t)2 = 2001 <=> 2ge -1188t + 12160 = 0

<=> t = 20. Khi d6 {m = 40 .

n =19

V~y gia tri 1611 nhAt ella A Iii 238 khi m = 40, n = 19.

88

89 ),

Bai 4.

Chung minh: a2 + b2 + c2 + 4abc <.!.. , 2

GQi S la dien tich tam giac, ta co:

1 1 1 1 2

S= 2 (2 -a)(2 -b)(2 -c) hay 16S = (1-2a)(l-2b)(1-2c) (1)

Vi 16S2 > 0 nen til (1) suy ra:

(1 "'- 2a)(1 - 2b)(1 - 2c) > 0

<=> 1 +4(ab+ be +ca)-8abc - 2(a+ b+c) > 0 <=> -1 +4(ab + be +ca)-8abc > O.

<=> -1 + 2[ (a+ b +C)2 _(a2 + b2 +c2) ] -8abc > 0 <=> 1-2(a2 + b2 +c2)-8abc > 0

<=> a2 + b2 +c2 +4abc <.!..

2

Chung minh: T= a2 + b2 + c2 + 4abc ~ Q , 27

Ta d~ dang chimg minh dtroc:

abc ~ (a + b -e)(b+c -a)(c+ a - b) (2) Til (2) ta co:

<=> abc ~(1- 2a)(1-2b)(1- 2c)

<=> abc ~ 1- 2(a + b + c)+4(ab + be +caj+Sabc <=> 9abc ~ -1 + 4(ab + be +ca)

1 4

<=> abc ~ --+-(ab+ bc+ ca) 9 9

Dodo:

2 4 . 16 5 2

T ~ (a+ b+c) -2(ab+ bc+ca)--+-(ab+ be-t-ea) = ---(ab+ bc+ca)

9 9 9 9

Ma (a-i-b+c)' ~3(ab+bc+ca)nen:

5 2 . 2 13

T~---(a+b+c) =-,

9 27· ,. 27

DAu " =" xay ra khi a = b = c =1/3,

TRllO'NG THPT CHuvEN nEN GIANG. TiNH nEN GIANG

Bai 1.

TIm gia tri nho nha't cua bi~u thirc:

as as a8

A = I 2 + 2 2 + .... + . n 2 '

(a2 +a2) (a2 +a2) (a2 +a2)

,I 2 2 3 n I

voi a., a2, ••• , an Ut nhii'ng s6 dirong va ala2 + a2a3 + .... + anal = K (K la hhng s6).

Bai 2.

TIm ta't ea cac s6 tt;t nhien n sao eho A = n200S + n2006 + n2 + n + 2 la mQt s6 nguyen t6.

Bai3.

1) Chang minh r~ng phirong trlnh sau eo 3 nghiernphan biet:

x3.,.... 3x + 1 = O. (*)

2) GQi XI , X2 , X3 la ba nghiern cila phuong trtnh (*) va ba di~m MI, M2, M3 eo hoanh dQ l~n IU<;1t la XI, X2, X3 n~m tren duongeong (C) eo phuong trlnh y = X4 - 6X2 + 4x + 6. Chung minh r~ng g6e toa dQ 0 la trong Him cua tam giac MIM2M3.

Bai 4.

BM 1.

Ap dung ba't d~ng thU'e Bu-nhi-a-cop-ski:

90

n

n

Ta cling co :

suy ra B ~

K2

V~y Min A = -, dat khi 4n

Bai 2.

Vdi n= 0 thi A = 32 Ia hop s5. Khi n = 1 thl A = 421a hop s5. Vdi n> 1 ta co:

A = (n2005 _ n) + (n2006 _ n2) + n2 + n + n2 + n + 2 = n(n2004 .... 1) + n2 (n2004 _ 1) + 2(n2 + n + 1).·

2(X)4 ( 3 )66K , 3 2

Don - 1 = n -1 chi a het cho rr - 1 = (n - 1) (n + n + 1) nen

n2(X)4 _ 1 chia he't cho n2 + n + 1 . Suy ra A chia he't cho n2 + n + 1 > 1.

V~y khong t6n tai s5 tt;t nhien n d~ A la s5 nguyen t5.

Bai 3.

1) f)~t f (x) = x3 - 3x + 1.

Ta co : f (- 2). f (0) <0 , f (0). f (1) < 0, f (1). f(2) < 0 nen phuong trlnh co 3 nghiern xi, X2, X3 va - 2 < XI < 0 < X2 < 1 < X3 < 2.

2) GQi G (xo, Yo) la trong tam tam giac MIM2M3.

Theo dinh li Vi-et: XI +X2 + X3 = 0 => Xu = 0

XI X2 + X2 X3 + XI X3 = - 3.

Vdi i = 1,2,3 taco:

x: -3xj +1= 0 => xt:::: x, (3xj -1) = 3x; -'Xj

4 4 4 3(·2 2 2) ( ..) 3( 2 2· 2)

=> XI +X2 +X3 = XI +X2 +X3 - XI +X2 +X3 = XI +X2 +XJ·

Ta co: YI +Y2 +Y3 =(x~ +x~ +x~)-6(x: +x; +xi)+4(xl +X2 +x1)+18 = -3(x; +x; +x;)+18=O.

Suy ra: y" = 0

V~y G == 0 nen g6c 0 la trong tam tam giac M,M2M3.

Bai 4.

Xet phuong trinh : Cos 4x = Cos 3x

<=> (cosx -1)( Scos'x +4cos2x -4cosx -1)= 0

r

<=> [cosx = 1

Scos'x + 4cos2x - 4cosx -1 = 0

Nh" h'" 2 21t 2 41t 2 61t an t ay : t, = cosT' t2 = cos?' tJ = cos?

lil 3 nghiern ciia PT : r' + t2 - 2t -1 = o.

{tl +t2 +tJ =-:-1 Theo Viet: tlt2 + ~tl + tit] =-2 tlt2t] -1

D~t: A = Vt: +Vt; +Vt;

B = .w, + Vt;t; + W.

Ta c6 : A1= 3AB-4 va B] = 3AB-5 suy ra A1s-1 =(3AB-4)(3AB-5)

~ (AB - 3 r + 7 :: 0 ~ AB = 3 - V7 ~ A] = 5-3.V7 ~ A = !./5-3 .V7.

TRUONG IHPT IHI xi CAO LANH. TiNH eONG THAP

Bai 1.

Duong phan giac trong va ngoai cua g6e C cua tam giac ABC cit dirong thang AB a L va M.Chmlg minh r~ng n~u C[= CM thi

. AC2 + BC2 = 4R 2 (R la ban kinh duong tron ngoai ti~p tam giac ABC).

92

93

).

Bai 2.

Cho a, b, c > O. Chung minh r~ng

111 a2006 + b2006 + C2006 ~

> 42006 ( 1 + 1 + 1 )

- (2a+b+c)2006 (a+2b+c)2006 . (a+b+2c)2006

Bai 3.

Cho tam giac ABC nhon thoa dieu kien:

1 + cos A cos B + cos B cos C + cos C cos A - (cos A + cos B + cos C)

= 2cosAcosBcosC.

Chirng minh r~ng tam giac ABC latam giac deu.

Bai 4.

Tim t~t ca cac nghiern nguyen (x.y) cua: (x ' + y)(x + y2) = (x - yi .

Hu'ONG DAN GIAI

Bail.

N€u CL = CM thi tam giac CML vuong din (do CL vuong goc CM, theo tinh chat hai duong phan giac cua Ig6c). Chon h~ true nhir hinh ve (0 trung diem ML), 0(0;0), A(a ; 0), B(b ; O),C(O ; c), L(c ; 0), M(-c ; 0).

Y

C

L B

M

x

Th lnh hA d' h'" . , AL . AC eo tin . c at irong p an giac: -. =--

LB CB

Ae AC2 (c-ai· a2 +c2. ,c2 (c2 J

__ =_. -¢:> = ¢:> b=-=> B -;0

LB2 CB2 (b-C)2 b2+C2 a a

Bai 2.

Ap dung B£>T Co-si, ta c6:

1 1

--+-->2

2006 2006 -

a b

(1)

Tirong tu:

1 1 22007

-. -+--> (2)

b2006 C2006 - (b+C)2006

1 1 22007

-+-> (3)

C2oo6 a2006 - (c+aiOO6

Cong (1 )(2)(3) vS theo v8, ta diroc:

_1_+_1_+_1_ > 22OO6[ 1 + 1 + 1 ] (4)

a 2006 b2OO6 C2006 - (a + b )2006 (b + C )2006 (c + a )2006

Tuong tu, ap dung (4) cho ta:

A>22006[ . 1 - 1 . IJ(5)

- (a+b+b+c)2006 + (b+c+c+a)2006 + (c+a+a+b)2006

k' All 1

Vul == + + .

(a+b)2006 (b+C)2006. (c+a)2006

Til (4), (5)~dpcm.

r•

94

Bai3.

De da cho diroc vi~t lai:

(1- cos A )(1- cos B)(I- cos C) = cos A cos B cosC . Ta chirng minh:

( 1 ~ cos A) ( 1- cos B). (1- cos C ) ~ 1

cosA cosB cosC

D~t: x = tg ~ ,y = tg ~ ,z = tg ~ (x, y, z > 0) .

" l-x2 l_y2. l-z2

The cOSA=--2 ,cOSB=--2 ,cOSC=--2 vao t lj ta diroc:

l+x l+y l+z

C =:' )C =~, )c =:' ) ~x~

ABC ¢:) tgAtgBtgC ~ cot g - cot g - cot g-

2. 2 2

ABC <=> tgA+ tgB +tgC ~ cotg-+,cotg-+cotg-

2 2 .2

B+C A+C A+B

<=> tgA + tgB + tgC ~ tg--+tg--+tg-- (2)

2 2 2

Ti~p thee: 0 < x < 1t ,0 < Y < 1t => tgx + tgy ~ 2tg x + y .

2 2 2

DAu = xay ra khi x = y. Ta co:

A-t-B tgA + tgB ~ 2tg-- 2

B+C tgB + tgC ~ 2tg -_ ..

2

A+C tgA + tgC ~ 2tg-- 2

Cong v~ thee v~ ta duoc (2). D~ng thirc 6 (2) xay ra khi A = B = C.

V~y tam giac ABC thoa dieu kien da cho la tam giac deu.

Bai 4.

De da cho diroc viet lai: y[2y2 + (x2 -3x)y + (x + 3x2)} = 0 (1) N~u y = 0 thi x bAt ki thuoc Z.

N~uy"# 0 : (1) => 2y2 + (x" -3x)y'+(x + 3x2) =0

D. = x(x3 -6x2 -15x-8) = x(x-8)(x+l)2

(I)

95 ,.

96

.:,.

D~ phirong trinh co nghiem nguyen thi L\ phai 1ft s6 chinh phuong, nrc la :

x(x -8) = i2 ¢:> (Ix - 41 ,-lal)(lx -41 +Ial) = 16

'{lx-41+lal~lx-41-lal A",

Do I I I I I I I I I I nen ta co cae tnrong hQ'P :

x-4+a+x-4-a=2x-4

{Ix -41+lal = 16 .

1) I I 11_ ~lx-41=8,5 (loai)

x-4-a-l .'

2) {Ix - 41 + lal = 8 ~ Ix _ 41 = 5 ~ [x = 9

Ix-41-lal=2 x=-1

3) .{Ix - 41 + lal = 4 ~ Ix _ 41 = 4 ~ [x = 8

, Ix - 41-lal = 4 x = 0

Thay vao phtrong trinh, ta co cac nghiem nguyen 1ft

(t ; 0) (t E Z),

(9; -6), (9; -21), (-1; -1), (8; -10).

TRU'CmG THPT IX SA sEc, tiNH SONG THAP

Bili 1.

TIm nghiern ciia h~ phirong trlnh:

, 1

(X2 +XY)(Y+2z)=g

3 x2 +y2 +3xy+4xz+2yz = - 4

x+y+z=o

(x c u-cy.z}.

f) -;

.

TIm nhii'ng e~p s6 nguyen (x, y) thoa phuong trlnh:

x3 -3x2 +3(y3 +1)x_(y3 +1)2 =0.

OLYMPIC MON TOAN - 7

97

Bai3.

Cho tam giac ABC, goi rna' m b' me theo thl1 tt,t la dQ dai trung tuyen xuat phat to' cac dinh A, B, C va r a' r b ' r c theo thu tt,t la ban kinh duC1ng tron bang tiep Ung v~i cac goc co dinh A, B, C. Chung minh r~ng:

2 2 2 2 2 2

ra + r b + r c ;::: m a + m b + me'

f)~ng thrrc xay ra khi nao?

Bai 4.

GQi I la Him duC1ng tron nQi tie'p tam giac ABC, duC1ng tron nay tie'p xuc vdi cac canh Be, CA, AB tai K, L, M tuong Ung. Qua B ke duC1ng thing song song vdi MK, duC1ng thing nay dt LM, LK tai S va R.

Chung minh goc RIS la goc nhon.

HUONG DAN GIAI

Bai 1.

f)~t u = x, v = x + y, t = Y + 2z suy ra uc v-c t.

1

uvt=-

8

=> -3

uv+vt+ tu =-

4

=> u, v, t la nghiem plnrong trlnh:4X 3 -3X = k

u+v-s te O

1

f)~t X = cos a => cos3 a = - (0< a < n ) 2

1t 2k1t

=> a = - + - vI u < v < t 9 3

71t => u=cos-

9

71t

x=cos-

9

51t 71t

y = cos--cos--

=> 99.

1t 51t . 71t 51t

Z = cos--cos--coS- = -cos-

9 9 9 9

51t 1t

V = cos-- t = cos-

9 9

Ta co:

S==(p-a)ra =(p-b)rb =(p-c)rc = ~p(p-a)(p-b)(p-c)

Dodo:

71t x=cos-

9

51t 71t

Y = cos - - cos-

9 9 la ba nghiem dn tim.

51t

z=-cos-

. 9

Bili 2.

THI : Ne'u y = - 1 thl ta dUQc phuong trmh:

x3 - 3x2 = 0 ~ x = 0 hay x = 3

" {x=o {X=3

Yay .

. y=-I' y=-1

TH2: Ne'u y *' -1 thl nhan 2 ve' p~u'dng trmh cho (y3 + 1) *' 0: (y3 + 1) x3 _ 3x2 (y3 + 1) + 3(l +1)2x - (l+I)3 = 0

~ x3l + (x_y3~1)3 = 0

~ yx = y3 + I-x

y3 +1 2

~ x=--=y -y+l y+l

V~y nghiem ciia phirong trmh ban d~U la :

{X = 0 ; {x = y2 - Y + 1

y=-l yeZ

BAi3.

S2 S2 S2

r2+r2+r2= 2+ 2+ 2

8 b c (p-a) (p-b) (p-c)

[(p- b)(p-c) (p-a)(p-c) (p- b)(p-a,)]

= p + + -=---'--=-~

p-a. p~b p-c

98

D~t {;::=:, suy ra {::~:: va x+ y+z v p .

z=p-c . y+z=a

Ta c~n chrrng minh:

(x+y+z{ ~:. ~ +"; ),,~[(y+z)' +(x+z)' +(x+y)'].

Tac6:

x2(r+~)+y2(~+~)+z2(X +I.)~2(X2 +y2 +Z2)

Z Y z x ,y x .

(x+y+z)(YZ + xz+ xy) ~ 2[X2 +yt +Z2]+xy+yz+xz X Y z

Theo ba't ding thrrc Co-si:

1 1

_(x2 + y2 +Z2) ~ -(xy+ yz+ xz)

2 2

Dod6:

VT~C~[ x2 +y2 +Z2 +Xy+yz+xz]~ ![(X+y)2 +(Y+Z)2 +(X+Z)2]

Ding thrrc xay ra khi x = y = z ~ a = b = c, hay tam giac ABC

d~u.

Bii 4.

De tha'y BI Ul true d6i xting cila MK hayBI .L MK ~ BI .L RS. Trong /). BKR, ta c6:

C

C B __ C BKcos-

BRK = -+- BKR = 90°-- ~BR = 2

2 2' 2 A

cos-

2

A BMcos-

Tirong ttl nhu v~y : BS = C 2

cos-

2

BR.BS = BM.BK= BK 2

~ IR 2 +IS 2 -RS 2= 2BI2 +BR 2 +BS 2 -(BR+BS) 2

= 2(BI 2 --' BR.Ss) = 2(BI 2 .: BK 2 ) = 2KI 2> o.

Theo dinh 11 eosin, ta suy ra rus < 90 ° .

Giai M bAt phirong trinh:

{6X2 • .JX3 -6x+5 =(x2 +2x-6)(X3 +4)

2 . 2

x+-~I+X x2

(1) (2)

TRUONG THPT BEN IRE. TiNH BEN IRE

Hai 1.

Cho tam giac ABC e6 cac g6e nhon nQi ti8p dirong tron tam 0 ban kinh R. GQi RJ, R2, R3 IAn lUQ11a ban kinh dirong tron ngoai ti~p cac tam giac OBC, OAC, OAB ; p la mra ehu vi cua tam giac ABC. Chung minh :

·729R7

RJR2R3 ~ 4'

16p

Hai 2.

Hai3.

GiAi phirong trinh : 3 [ 1 + cos;x ]. + 4tg'x = 7 .

(nQ cos x

l7 Tim nghiem nguyen duong cua phuong trinh :

2x 7 + 5x4y2 + 7X4 - 2x3y - 5y3 -7y = 2006 .

HUONG DAN GUI

BBi 1.

f)~t BC = a , AC = b , AB = e , SABe = S va p l~nira ehu vi tam giac ABC.

R a.R2

=> )=-- 4S0BC

b.R2 e.R2

Tuong t1;l: R2 = --. _. ; R3 =--

4S0AC . 4S0AB

T 6' S _ OB.OC.BC =_R2_a

ae. OBC - 4R) 4R)

100

101

R R R = R6.abe,~ R6abe

=> I 2 3 648' S ' 'S ' ( )3 '

, • OBC· OAC· OAB '. 64 SOBC + S~AC + SOAB

R R R 27R6abe _ 27R'

=> > ---

I 2 3 - 64S3 1682

, l

2 (p-a+p'-b+p-e) p4

Mel S = p(p - a )( p -- b )( p - e ) ~ p " =-' ,

, '3 ' 27

, A 729R'

nen: RIR2R3 ~ , 4 •

16p

Ding thirc xay ra khi vel chi khi tam giac ABC d~u.

OBi 2.

'Til (2) ta suy ra: x + ~ > 1 => x > 0 => x3 + 4 > 0 . Do d6: x

[x> -1 +.J7 t:

x2 + 2x..,..6 > 0 ~ ," t: => x > ~7 -1

x<-I-v7

K~t hQ"P di~uki~n x3 - 6x + 5 ~ 0 ta dtroc:

x ~ -I+m. (3) 2

Khi d6 taldn lugt e6 cae nl$1 xet sau:

, " I" 1

~x 3 - 6x + 5 = ~ (x -1)( x 2 + X - 5) ~ 2 ( x -1 + ,x 2 + X - 5) = 2" ( X 2 + 2x - 6 )

.r: IT- 1 (Xl x3) i . ,-J

X2=VX6 = ~- -+-, +4=-(x3+4)

3 2 2 3 '

til d6 ta suy ra:

6x2~x3 - 6x + 5 ~ (x2 + 2x -6)(x3 + 4).

V~y (1) nrong dirong voi:

{x-I = x2 +x-5

x3 = 4 ~ x = 2 , thea man (2).

2 " .

V~y bAt phuong trinh c6 nghiem ~ = 2.

Phuong trinh da eho nrong duong: (x4_y). (5y2 +2x3 +7)=1.2.17.59

Do (x4_y) >0 => x ~ 2

=> 5y2+2x3+7~5+16+7=28 =>(x4_y) e {1;2;17;34;59}

[16-y = 1 _[y = 15

Neu x = 2 thi --

16-y=2 y=14

Chi e6 e~p (2; 14) thea phirong trinh. Neu x ~ 3:

59 ~ (x4_y) ~ (81-y)=>y~22

5y2 + 2x3 + 7 > 2006 : khong thoa phirong trinh V~y phirongtrinh e6 nghiem duy nhAt (2; 14 ).

Bai3.,

DiU U= eos2x +1 vav=tg2x.

. cosix

Phuong trinh e6 dang: 3u4 +4V3 = 7. Ta e6:

, eos2x (1 2) cos2x + 1 2eos2x 2

u+v=--+ +tg x = , =

, ' cos'x cos'x cos'x

Sir dung bAt ding thirc Co-si ta e6:

{u4 +1,,+1+1 ~ 4u {3(U4 +3) ~ 12u - 4 3

=> =>3u +4v ~7

v3 +1+1 ~ 3v 4(v3 +2) ~ 12v .

I. ' {u4 = 1 ,7t

Dau bang xay ra ¢:::> ¢:::> v = 1 ¢:::> tgx = ± 1 ¢:::> x = ± - + kn

v' =1 4

V~yphuongtrinhe6nghi~m: x =±: +k7t (keZ).

Bai4.

102

TRu'O'NG THPJ LtJ'U VAN KIET. tiNH ViNH LONG

Hili 1.

a) Tim nghiem nr nhien cua phuongtrinh:

(X2 + 4y+ 28 i = 17( X4 + y4 + 14y2 + 49) .

. b) Tim cac gia tri nguyen dirong khac nhau XI, X2, ... , Xn sao cho:

1 1 1

-2 +-2 +"'+-2 = 1.

XI x2 xn

Hili 2~

a) Gia su phuong trinh X4 + bx3 + CX2 + bx + 1 = 0 c6 nghiem, Chirng minh : b2 + ( C - 2 i > 3.

b) Giai phirong trinh : X3 + 3x - 3 = O.

Hili 3.

Tim cac ham s6 f(x) va g(x) xac dinh boi M sau :

{f(X -1) + g(2x + 1) = 2x

f(2x + 2) + 2g( 4x + 7) = x-I .

Hili 4.

a) Cho ur giac ABCD nQi ti~p trong (0, R). Ve Ax vuong g6c AD dt BC tai E, ve Ay vuong g6c AB c~t CD tai F. Chung minh EF qua 0.

b) Cho tam giac ABC c6 dinh, ve hinh thoi BCDE. TiT D, E ve dirong vuong g6c AB va AC, cac dirong nay c~t nhau tai M. Tim quy tich di~m M.

Hu'O'NG DAN GIAI

Hili 1.

a) Ap dung BDT Bu-nhi-a-cop-xki :

[l.x2 + 4.(y2 + 7)J s (12 + 42)[ X4 + (y2 + 7)2 ] ~17(x4 +y4+14y2 +49).

Do d6 4x2 = i + 7 ~ (2x + y)( 2x - y ) = 7. Vi x, YEN nen 2x + y ~ 2x _- y ~ 0 .

a) Ta c6 X4 + bX3 + CXl + bx + 1 = 0 <=!> X2 + bx + c + b + -;. = 0 x x

1 ' 1

<=!> X2 +-2 +b{x+-)+c=O

x x

Ta c6 e + bt + c- 2 = 0 voi t = x+..!_ => x2 + -;. = t2 - 2' va It I ~ 2 "xx'

<=!>t2 ={2-c)-bt

Ap dung BBT Bu-nhi-a-cep-xki:

t4 = [(2-c).l-b.tt S[(2-C)2 +b2](t2'+I)

t4

-- S (2 - C)2 + b2

t2 + 1 ' ,

1 ' '

Ma Xl + -2 ~ 2 => e ~ 4 => t4 ~ 16 nen

x

t4 -3{e + 1) = t4 -3t2 -3 ~16-12-31 > 0

, t4

t4 > 3{t2 + 1) <=!> -2- >3 t +1

V~y : b2 + ( C - 2 i >3.

, 1

b) B~t x = y--

Y

1 3 1· . 3 1

Ta diroc (y--) -3(y --)-3= 0 ~ y --~3::: O.

y Y y3

{2X + y = 7 {x = 2

Ta c6 <=!>. V~y S = {{2,3)} .

2x-y =1 y =3

b) Ta c6 xi, Xl, ... ,Xn khong be han 2 nen:

11 1<11 1,11 1

2+-2 +"'+-2 --2 +2"+"'+ 2 <--+-+ ... +

XI X2 ' xn 2 3 (n+l) 1.2 2.3 nfn+ I)

<1_.!.+l_!+ ... +! I_,. =1 __ 1_ <1

2 2, 3 n n+ l n+I

VA 1 1 1 1

~y -2 +--;:+"'+-2 < .

XI ' X2 xn

Do d6, khong c6 cac gia tringuyen duongkhacnhau thea man:

1 1 1

-2 +-2 +"'+-2 =1.

XI x2 xn

Oai 2.

104

D~t t:::: y3 , ta co t'-! - 3 = 0 t

e - 3t -1 = 0 <=> t = ~ (3 ±.Ji3) :::) y = 3 ~ (3 ±.Ji3)

V~YX;=3.!._(3±,Ji3)-~ 2d. 23±-.;13

Bai3.

D~tx-l =2u+2:::)x=2u+3,2x::::4u+6

f(x -1) + g(2x + 1) = 2x :::) f(2u + 2) + g(4u+ 7) == 4u +6 :::) f(2x + 2)+ g(4x + 7) =4x + 6

. , {.f(2X+2)+g(4X+7)=4X+6 {f(2.X+2)=7X+13

Khi do ta co·· :::)

f(2x + 2)+ 2g(4x + 7) = x-I g(4x + 7) = -3>C!-7

u-2 7

Dat u = 2x + 2 :::) x = _. - :::) f'(x) = - x + 6

. 2· 2

. t-7 . 3 7

DiU t = 4x + 7 :::) x = -- :::) g(x) =.--x--

. 4 4 4

Bai 4.

a) Thirc hien DEF: A H M Chung minh M E (0). Taco:

BcD + BAD = 1800

rna EAF + BAi);,1800 nen

BcD=W"

----- ----- ----------

Vi EAF = EMF nen BCD = EMF

Suy ra til giac EFCM nQi tit~p, suy ra:

McE=MFE,

--- ---

MFE=EFA

--- .-

EFA=MAB.

M

-- .-

Do do MCE = MAB :::)

ABMC nQi ti€p, suy ra EF nam tren dtrong kinh hay EF quaO.

b) GQi H la tnrc tam cua tam giac ABC, d~t BC = a.

,.,

105

Potrebbero piacerti anche